听力首页 听力教程 VOA慢速 英语歌曲 外语下载 英语小说 英语词典 在线背单词 听力论坛 韩语学习
听力专题 英语教材 VOA标准 英语动画 英语考试 资源技巧 英语翻译 单词连连看 听力家园 德语学习
听力搜索 英语导读 BBC英语 英语视频 英语电台 英语QQ群 外语歌曲   英语游戏 英语网刊 日语学习
当前位置: 英语听力论坛 » 英语四六级 » 王长喜-六级考试标准阅读60篇(51-60)
返回列表 发帖

王长喜-六级考试标准阅读60篇(51-60)

第51篇:(Unit 13,Passage 3)

With the start of BBC World Service Television, millions of viewers in Asia and America can now watch the Corporation’s news coverage, as well as listen to it.

And of course in Britain listeners and viewers can tune into two BBC television channels, five BBC national radio services and dozens of local radio stations. They are brought sport, comedy, music, news and current affairs, education, religion, parliamentary coverage, children’s programs and films for an annual license fee of 83 per household.

It is a remarkable record, stretching back over 70 years—yet the BBC’s future is now in doubt. The Corporation will survive as a publicly-funded broadcasting organization, at least for the time being, but its role, its size and its programs are now the subject of a nationwide debate in Britain.

The debate was launched by the government, which invited anyone with an opinion of the BBC—including ordinary listeners and viewers—to say what was good or bad about the Corporation, and even whether they thought if it was worth keeping. The reason for its inquiry is that the BBC’s royal charters runs out in 1996 and it must decide whether to keep the organization as it is or to make changes.

Defenders of the Corporation—of whom there are many—are fond of quoting the American slogan “If it ain’t broke, don’t fix it.” The BBC “ain’t broke”, they say, by which they mean it is not broken (as distinct from the word “broke”, meaning having no money), or why bother to change it?

Yet the BBC will have to change, because the broadcasting world around it is changing. The commercial TV channels—ITV and Channel 4—were required by the Thatcher Government’s Broadcasting Act to become more commercial, competing with each other for advertisers, and cutting costs and jobs. But it is the arrival of new satellite channels—funded partly by advertising and partly by viewers’ subscriptions—which will bring about the biggest change in the long term.

1.The world famous BBC now is confronted with ___.
A.the problem of news coverage
B.an uncertain prospect
C.inquiries by the general public
D.shrinkage of audience

2.In the passage, which of the following about the BBC is not mentioned as the key issue?
A.Extension of its TV service to Far East.
B.Programs as the subject of a nation-wide debate.
C.Potentials for further international co-operations.
D.Its existence as a broadcasting organization.

3.The BBC’s “royal charter” (Paragraph 4) represents ___.
A.the financial support from the royal family
B.the privileges granted by the Queen
C.a contract with the Queen
D.a unique relationship with the royal family

4.The word “broke” in “If it ain’t broke, don’t fix it” means ___.
A.broke down
B.bankrupt
C.fragmented
D.penniless

5.The first and foremost reason why the BBC has to read just itself is no other than ___.
A.the emergence of commercial TV channels
B.the enforcement of Broadcasting Act by the government
C.the urgent necessity to reduce cost—and—job expenses
D.the challenges of new satellite channels

第51篇答案:BCCDD


第52篇:(Unit 13,Passage 4)


Federal Reserve System, central banking system of the United States, popularly called the Fed. A central bank serves as the banker to both the banking community and the government; it also issues the national currency, conducts monetary policy, and plays a major role in the supervision and regulation of banks and bank holding companies. In the U.S. these function are the responsibilities of key officials of the Federal Reserve System: the Board of Governors, located in Washington, D.C., and the top officers of 12 district Federal Reserve banks, located throughout the nation. The Fed’s actions, described below, generally have a significant effect on U.S. interest rates and, subsequently, on stock, bond, and other financial markets.

The Federal Reserve’s basic powers are concentrated in the Board of Governors, which is paramount in all policy issues concerning bank regulation and supervision and in most aspects of monetary control. The board enunciates the Fed’s policies on both monetary and banking matter. Because the board is not an operating agency, most of the day-to day implementation of policy decisions is left to the district Federal Reserve banks, stock in which is owned by the commercial banks that are members of the Federal Reserve System. Ownership in this instance, however, does not imply control; the Board of Governors and the heads of the Reserve banks orient their policies to the public interest rather than to the benefit of the private banking system.

The U.S. banking system’s regulatory apparatus is complex; the authority of the Federal Reserve is shared in some instances for example, in mergers or the examination of banks with other Federal Deposit Insurance Corporation (FDIC). In the critical area of regulating the nation’s money supply in accordance with national economic goals, however, the Federal Reserve is independent within the government, Income and expenditures of the Federal Reserve banks and of the board of governors are not subject to the congressional appropriation process; the Federal Reserve is self-financing. Its income ($20.2 billion in 1992) comes mainly from Reserve bank holdings of income-earning securities, primarily those of the U.S. government. Outlays ($1.5 billion in 1992) are mostly for operational expenses in providing services to the government and for expenditures connected with regulation and monetary policy. In 1992 the Federal Reserve returned 416.8 billion in earnings to the U.S. treasury.

1.The Fed of the United States ___.
A.function as China Bank
B.is the counterpart of People’s Bank of China
C.is subjected to the banking community and government
D.has 13 top officers who can influence the American financial market

2.The fact that stock in the Fed belongs to commercial banks ___.
A.doesn’t mean the latter is in control
B.means the latter is in control
C.means the latter is subjected to the Reserve banks
D.means the Reserve banks orient the latter’s policies

3.Which of the following statements is not true according to the passage?
A.The fed is a very big, complex and significant system which comprises many local banks.
B.All the commercial banks are not the components of Federal Reserve System.
C.Board of governors is the supreme policy-makers of America.
D.District Reserve banks rather than Board of governors perform the day-to-day policies.

4.The authority of the federal Reserve ___.
A.has to be shared with other establishments.
B.is exclusive at other times
C.isn’t limited by comptroller of the Currency and FDIC
D.is limited by Board of governors

5.Income of the Board of governors ___.
A.is borrowed from the U.S. treasury
B.is used by the government to make various policies
C.comes from the U.S. Treasury
D.is not granted by the government

第52篇答案:BACBD

王长喜-六级考试标准阅读60篇(51-60)

第53篇:(Unit 14,Passage 1)

The food irradiation process is a simple one. The new U.S. plant, Vindicator of Florida Incorporated in Mulberry, Fla., uses a material called cobalt 60 to irradiate food. Cobalt 60 is radioactive isotope (form) of the metallic element cobalt. Cobalt 60, which gives off radiation in the form of gamma rays, is also used for radiation therapy for cancer patients and for sterilizing hospital equipment. The radioactive isotope is created by bombarding cobalt with subatomic particles in a nuclear reactor. However, irradiation plants do not themselves contain nuclear reactors.

In the irradiation plant, food is exposed to thin rods of cobalt 60. The rods give off gamma rays, which disrupt chemical processes in contaminating organisms. The disruption breaks down the cell walls of organisms or destroys their genetic material. The dose, set by the U.S. Food and Drug Administration (FDA), is enough to kill organisms on food, but not enough to produce significant changes in the food itself.

Although irradiation slightly decreases the nutritive value of foods, the loss is less than that produced by some other methods of food preservation. Canning, for example, results in a much greater loss of nutrients.

Those who object to irradiation say that the process may create substances not found in nonirradiated food. Since the 1960’s researchers have studied irradiated food at microscopic levels to try to find such substances, called unique radiolytic products. After reviewing these studies, the FDA determined that compounds formed during irradiation are similar to substance found in nonirradiated foods and are not dangerous to consume.

Destruction of microorganisms that cause illness is an important goal of irradiation. About 250 million cases of food poisoning or 1 per person—occur every year in the U.S., according to FDA estimates. Food poisoning can cause vomiting, diarrhea, fever, headache—and, occasionally, death.

Because of the apparent safety of food irradiation, and the problems presented by contaminated food, scientific groups—including the American Medical Association, the World Health Organization, and the United Nations food and Agriculture Association—have voiced nearly universal support for the process. Worldwide, 38 nations have approved irradiation for 355 products.

Like microwave ovens, food irradiation has aroused apprehension and misunderstanding. Yet it has been scrutinized more thoroughly than other methods of food treatment that we have come to regard as safe, and it appears to be a method whose time has come.

1.Cobalt 60, besides irradiating food, is also employed to ___.
A.detect metallic flaws
B.run a nuclear reactor
C.cure cancer patients
D.strengthen concrete walls

2.Gamma rays used to irradiate food ___.
A.are generally not strong enough to destroy contaminating organisms
B.do not bring about significant changes in the food itself
C.may destroy some of the nutrients in the food
D.should be submitted to FDA for approval

3.Irradiated food ___.
A.certainly loses its nutritive value
B.maintains its nutritive value no different from the nonirradiated
C.keeps its nutritive value better than canned food
D.is recommended as the best of all preserved foods

4.With cases of food poisoning increasing, ___.
A.food irradiation should be carried out with care
B.it is more urgent to irradiate foods
C.medical researches into treatment of the diseased should be strengthened
D.Americans are beginning to accept food irradiation

5.The passage may be taken from ___.
A.a news report
B.a textbook of food processing
C.a book of popular science
D.a manual of food irradiation

第53篇答案:CBCCD


第54篇:(Unit 14,Passage 2)

Until recently, women in advertisements wore one of three things—an apron, a glamorous dress or a frown. Although that is now changing, many women still feel angry enough to deface offending advertisements with stickers protesting, “This ad degrades women.” Why does this sort of advertising exist? How can advertisers and ad agencies produce, sometimes, after months of research, advertising that offends the consumer?

The Advertising Standards Authority (the body which deals with complaints about print media) is carrying out research into how women feel about the way they are portrayed in advertisements. Its conclusions are likely to be what the advertising industry already knows: although women often irritated by the way they are seen in ads, few feel strongly enough to complain.

Women are not the only victims of poor and boring stereotypes—in many TV commercials men are seen either as useless, childish oafs who are unable to perform the simplest household tasks, or as in considerate boors, permanently on the lookout for an escape to the pub. But it is women who seem to bear the brunt of the industry’s apparent inability to put people into an authentic present-day context.

Yet according to Emma Bennett, executive creative director of a London advertising agency, women are not infuriated by stereotypes and sexist advertising. It tends to wash over them, they are not militant or angry—they just find it annoying or tiresome. They reluctantly accept outdated stereotypes, but heave a sigh of relief when an advertisement really gets it right.
She says that it is not advertising’s use of the housewife role that bothers women, but the way in which it is handles. “Researchers have often asked the wrong questions. The most important thing is the advertisement’s tone of voice. Women hate being patronized, flattered or given desperately down-to-earth commonsense advice.”

In the end, the responsibility for good advertising must be shared between the advertiser, the advertising agency and the consumer. Advertising does not set trends but it reflects them. It is up to the consumer to tell advertisers where they fail, and until people on the receiving end take the business seriously and make their feelings known, the process of change will remain laboriously slow.

1.Despite recent changes in attitudes, some advertisements still fail to ___.
A.change women’s opinions of themselves
B.show any understanding of people’s feelings
C.persuade the public to buy certain products
D.meet the needs of the advertising industry

2.According to the writer, the commonest fault of present day advertising is to ___.
A.condemn the role of the housewife
B.ignore protests about advertisements
C.present a misleading image of women
D.misrepresent the activities of men

3.Research suggests that the reaction of women towards misrepresentation by advertisement is ___.
A.apathy
B.hostility
C.approbation
D.unbelief

4.Emma Bennett suggests that advertisement ought to ___.
A.give further emphasis to practical advice
B.change their style rather than their content
C.use male images instead of female ones
D.pay more compliments to women than before

5.Ultimately the advertising industry should ___.
A.take its job more earnestly
B.do more pioneering work
C.take notice of the public opinion
D.concentrate on the products advertised.

第54篇答案:BCABC

TOP

王长喜-六级考试标准阅读60篇(51-60)

第55篇:(Unit 14,Passage 3)

Pronouncing a language is a skill. Every normal person is expert in the skill of pronouncing his own language; but few people are even moderately proficient at pronouncing foreign languages. Now there are many reasons for this, some obvious, some perhaps not so obvious. But I suggest that the fundamental reason why people in general do not speak foreign languages very much better than they do is that they fail to grasp the true nature of the problem of learning to pronounce, and consequently never set about tackling it in the right way. Far too many people fail to realize that pronouncing a foreign language is a skill—one that needs careful training of a special kind, and one that cannot be acquired by just leaving it to take care of itself. I think even teachers of language, while recognizing the importance of good accent, tend to neglect, in their practical teaching, the branch of study concerned with speaking the language. So the first point I want to make is that English pronunciation must be taught; the teacher should be prepared to devote some of the lesson time to this, and by his whole attitude to the subject should get the student feel that there is a matter worthy of receiving his close attention. So, there should be occasions when other aspects of English, such as grammar or spelling, are allowed for the moment to take second place.

Apart from this question of the time given to pronunciation, there are two other requirements for the teacher: the first, knowledge; the second, technique.

It is important that the teacher should be in possession of the necessary information. This can generally be obtained from books. It is possible to get from books some idea of the mechanics of speech, and of what we call general phonetic theory. It is also possible in this way to get a clear mental picture of the relationship between the sounds of different languages, between the speech habits of English people and those, say, of your students. Unless the teacher has such a picture, any comments he may make on his students pronunciation are unlikely to be of much use, and lesson time spent on pronunciation may well be time wasted.

1.What does the writer actually say about pronouncing foreign languages?
A.Only a few people are really proficient.
B.No one is really an expert in the skill.
C.There aren’t many people who are even fairly good.
D.There are even some people who are moderately proficient.

2.The writer argues that going about the problem of pronunciation in the wrong ways ___.
A.an obvious cause of not grasping the problem correctly
B.a fundamental consequence of not speaking well
C.a consequence of not grasping the problem correctly
D.not an obvious cause of speaking poorly

3.The best way of learning to speak a foreign language, he suggests, is by ___.
A.picking it up naturally as a child
B.learning from a native speaker
C.not concentrating on pronunciation much
D.undertaking systematic work

4.The value the student puts on correct speech habits depends upon ___.
A.how closely he attends to the matter
B.whether it is English that is being taught
C.his teacher’s approach to pronunciation
D.the importance normally given to grammar and spelling

5.How might the teacher find himself wasting lesson time?
A.By spending lesson time on pronunciation.
B.By making ill-informed comments upon pronunciation.
C.By not using books on phonetics in the classroom.
D.By not giving students a clear mental picture of the different between sounds.

第55篇答案:CCDCB


第56篇:(Unit 14,Passage 4)

Work is a very important part of life in the United States. When the early Protestant immigrants came to this country, they brought the idea that work was the way to God and heaven. This attitude, the Protestant work ethic, still influences America today. Work is not only important for economic benefits, the salary, but also for social and psychological needs, the feeling of doing something for the good of the society. Americans spend most of their lives working, being productive. For most Americans, their work defines them; they are what they do. What happens, then when a person can no longer work?

Most Americans stop working at age sixty-five or seventy and retire. Because work is such an important part of life in this culture, retirement can be very difficult. Retirees often feel that they are useless and unproductive. Of course, some people are happy to retire; but leaving one’s job, whatever it is a difficult change, even for those who look forward to retiring. Many retirees do not know how to use their time or they feel lost without jobs.

Retirement can also bring financial problems. Many people rely on Social Security checks every month. During their working years, employees contribute a certain percentage of their salaries to the government. When people retire, they receive this money as income. These checks do not provide enough money to live on, however, because prices are increasing very rapidly. Senior citizens, those over sixty-five, have to have savings in the bank or other retirement plans to make ends meet. The rate of inflation is forcing prices higher each year; Social Security checks alone cannot cover Medicare (health care) and welfare (general assistance) but many senior citizens have to change their lifestyles after retirement. They have to spend carefully to be sure that they can afford to but food, fuel, and other necessities.

Of course, many senior citizens are happy with retirement. They have time to spend with their families or to enjoy their hobbies. Some continue to work part time; others do volunteer work. Some, like those in the Retired Business Executives Association, even help young people to get started in new business. Many retired citizens also belong to “Golden Age” groups. These organizations plan trips and social events. There are many opportunities for retirees.

Americans society is only beginning to be concerned about the special physical and emotional needs of its senior citizens. The government is taking steps to ease the problem of limited income. They are building new housing, offering discounts in stores and museums and on buses, and providing other services, such as free courses, food service, and help with housework.

Retired citizens are a rapidly growing percentage of the population. This part of the population is very important and we must respond to their needs. After all, every citizen will be a senior citizen some day.

1.The early immigrants considered work ___.
A.too hard
B.important
C.pleasant
D.dull

2.Why do Americans like working? Because working ___.
A.doesn’t only mean money but it is also psychological
B.can make life more comfortable
C.can prove people to be independent
D.gives people funny

3.We can safely put forward that retirees who ___.
A.have no financial problems still want to earn more money
B.have financial problems still feel lost
C.have no financial problems still feel lost
D.have no financial problems feels it’s hard to make ends meet

4.According to the passage the government ___.
A.hadn’t paid attention to the retirees’ problems
B.has already solved a lot of retirees’ problems
C.has just begun to pay attention to the retirees’ problems
D.won’t pay attention to the retirees’ problems

5.Which of the following is not steps taken for the benefit of senior citizens by the government?
A.New housing has been built.
B.The old are offered discounts in stores.
C.Senior citizens are provided free courses, food service.
D.None.

第56篇答案:BACCD

TOP

王长喜-六级考试标准阅读60篇(51-60)

第57篇:(Unit 15 ,Passage 1)

If we look at education in our own society, we see two sharply different factors. First of all, there is the overwhelming majority of teachers, principals, curriculum planners, school superintendents, who are devoted to passing on the knowledge that children need in order to live in our industrialized society. Their chief concern is with efficiency, that is, with implanting the greatest number of facts into the greatest possible number of children, with a minimum of time, expense, and effort.

Classroom learning often has as its unspoken goal the reward of pleasing the teacher. Children in the usual classroom learn very quickly that creativity is punished, while repeating a memorized response is rewarded, and concentrate on what the teacher wants them to say, rather than understanding the problem.

The difference between the intrinsic and the extrinsic aspects of a college education is illustrated by the following story about Upton Sinclair. When Sinclair was a young man, he found that he was unable to raise the tuition money needed to attend college. Upon careful reading of the college catalogue, however, he found that if a student failed a course, he received no credit for the course, but was obliged to take another course in its place. The college did not charge the student for the second course, reasoning that he had already paid once for his credit. Sinclair took advantage of this policy and not a free education by deliberately failing all his courses.

In the ideal college, there would be no credits, no degrees, and no required courses. A person would learn what he wanted to learn. A friend and I attempted to put this ideal into action by starting a serials of seminars at Brandeis called “Freshman Seminars Introduction to the Intellectual Life.” In the ideal college, intrinsic education would be available to anyone who wanted it—since anyone can improve and learn. The student body might include creative, intelligent children as well as adults; morons as well as geniuses (for even morons can learn emotionally and spiritually). The college would be ubiquitous—that is, not restricted to particular buildings at particular times, and teachers would be any human beings who had something that they wanted to share with others. The college would be lifelong, for learning can take place all through life. Even dying can be a philosophically illuminating, highly educative experience.

The ideal college would be a kind of education retreat in which you could try to find yourself; find out what you like and want; what you are and are not good at. The chief goals of the ideal college, in other words, would be the discovery of identity, and with it, the discovery of vocation.

1.In the author’s opinion, the majority of education workers ___.
A.emphasize independent thought rather than well-memorized responses
B.tend to reward children with better understanding rather than with a goal for credits
C.implant children with a lot of facts at the expense of understanding the problem
D.are imaginative, creative and efficient in keeping up with our industrialized society

2.Children in the usual classroom learn very quickly when ___.
A.they are required to repeat what teacher has said
B.they read books that are not assigned by the teacher
C.they know how to behave themselves in face of the teacher
D.they can memorize the greatest number of facts in the shortest period of time

3.An extrinsically oriented education is one that ___.
A.focuses on oriented education
B.takes students’ need into account
C.lays emphases on “earning a degree”
D.emphasizes learning through discussion

4.To enter the author’s ideal college, a student ___.
A.has to pass an enrollment exam
B.should be very intelligent
C.needn’t worry about homework
D.can be best stimulated for creative work

5.The author’s purpose of writing the article is ___.
A.to advocate his views
B.to criticize college students
C.to stress self-teaching attitude
D.to put technological education to a later stage

第57篇答案:CACCA


第58篇:(Unit 15 ,Passage 2)

Culture is the total sum of all the traditions, customs, beliefs, and ways of life of a given group og human beings. In this sense, every group has a culture, however savage, undeveloped, or uncivilized it may seem to us.

To the professional anthropologist, there is no intrinsic superiority of one culture over another, just as to the professional linguist there is no intrinsic hierarchy among languages.
People once thought of the languages of backward groups as savage, undeveloped forms of speech, consisting largely of grunts and groans. While it is possible that language in general began as a series of grunts and groans, it is a fact established by the study of “backward” languages that no spoken tongue answers that description today. Most languages of uncivilized groups are, by our most severe standards, extremely complex, delicate, and ingenious pieces of machinery for the transfer of ideas. They fall behind our Western languages not in their sound patterns or grammatical structures, which usually fully adequate for all language needs, but only in their vocabularies, which reflects the objects and activities known to their speakers. Even in this department, however, two things are to be noted: 1. All languages seem to possess the machinery for vocabulary expansion, either by putting together words already in existence or by borrowing them from other languages and adapting them to their own system. 2. The objects and activities requiring names and distinctions in “backward” languages, while different from ours, are often surprisingly numerous and complicated. An accidental language distinguishes merely between two degrees of remoteness (“this” and “that”); some languages of the American Indians distinguish between what is close to the speaker, or to the person addressed, or removed from both, or out of sight, or in the past, or in the future.

This study of language, in turn, casts a new light upon the claim of the anthropologists that all culture are to be viewed independently, and without ideas of rank or hierarchy.

1.the language of uncivilized groups as compared to Western languages are limited in ___.
A.sound patterns
B.vocabularies
C.grammatical structures
D.both A and B

2.The author says that professional linguists recognize that ___.
A.Western languages are superior to Eastern languages
B.All languages came from grunts and groans
C.The hierarchy of languages is difficult to understand
D.There is no hierarchy of languages

3.The article states that grunt-and-groan forms of speech are found ___.
A.nowhere today
B.among the Australian aborigines
C.among Eastern cultures
D.among people speaking “backward” languages

4.According to the author, languages, whether civilized or not, have ___.
A.the potential for expanding vocabulary
B.their own sound patterns
C.an ability to transfer ideas
D.grammatical structures

5.Which of the following is implied but not articulated in the passage?
A.The study of languages has discredited anthropological studies.
B.The study of language has reinforced anthropologists in their view that there is no hierarchy among cultures.
C.The study of language is the same as the study of anthropologists.
D.The study of languages casts a new light upon the claim of anthropologists.

第58篇答案:BDAAB

TOP

王长喜-六级考试标准阅读60篇(51-60)

第59篇:(Unit 15 ,Passage 3)

Most people would probably agree that many individual consumer adverts function on the level of the daydream. By picturing quite unusually happy and glamorous people whose success in either career of sexual terms, or both, is obvious, adverts construct an imaginary world in which the reader is able to make come true those desires which remain unsatisfied in his or her everyday life.

An advert for a science fiction magazine is unusually explicit about this. In addition to the primary use value of the magazine, the reader is promised access to a wonderful universe through the product—access to other mysterious and tantalizing worlds and epochs, the realms of the imagination. When studying advertising, it is therefore unreasonable to expect readers to decipher adverts as factual statements about reality. Most adverts are just too meagre in informative content and too rich in emotional suggestive detail to be read literally. If people read then literally, they would soon be forced to realize their error when the glamorous promises held out by the adverts didn’t materialize.

The average consumer is not surprised that his purchase of the commodity does not redeem the promise of the advertisement, for this is what he is used to in life: the individual’s pursuit of happiness and success is usually in vain. But the fantasy is his to keep; in his dream world he enjoys a “future endlessly deferred”.

The Estivalia advert is quite explicit about the fact that advertising shows us not reality, but a fantasy; it does so by openly admitting the daydream but in a way that insists on the existence of a bridge linking daydream to reality—Estivalia, which is “for daydream believers”, those who refuse to give up trying to make the hazy ideal of natural beauty and harmony come true.

If adverts function on the daydream level, it clearly becomes in adequate to merely condemn advertising for channeling readers’ attention and desires towards an unrealistic, paradisiacal nowhere land. Advertising certainly does that, but in order for people to find it relevant, the utopia visualized in adverts must be linked to our surrounding reality by a casual connection.

1.The people in adverts are in most coves ___.
A.happy and glamorous
B.successful
C.obvious
D.both A and B

2.When the glamorous promises held out by the adverts didn’t materialize the average consumer is not surprised, because ___.
A.The consumer is used to the fact that the individual’s pursuit of happiness and success is usually in vain.
B.Adverts are factual statements about reality.
C.The consumer can come into the realms of imagination pictured by adverts.
D.Adverts can make the consumer’s dreams come true.

3.What’s the bridge linking daydream to reality in adverts?
A.The product.
B.Estivalia.
C.Pictures.
D.Happy and glamorous people.

4.Why does the consumer accept the daydream in adverts?
A.Because the consumer enjoys a “future endlessly deferred.”
B.Because the consumer gives up trying to make his dream come true.
C.Because the utopia is visualized in adverts.
D.Because his purchased of the commodity does not redeem the promise of the advertisement.

5.What is this passage mainly concerned with?
A.Many adverts can be read literally.
B.Everyone has a daydream.
C.Many adverts function on the level of the daydream.
D.Many adverts are deceitful because they can not make good their promises.

第59篇答案:DABAC


第60篇:(Unit 15 ,Passage 4)

The establishment of the Third Reich influence events in American history by starting a chain of event, which culminated in war between Germany and the United states. The complete destruction of democracy, the persecution of Jew, the war on religion, the cruelty and barbarism of the Nazis, and especially, the plans of Germany and her allies, Italy and Japan, for world conquest caused great indignation in this country and brought on fear of another world war. While speaking out against Hitler’s atrocities, the American people generally favored isolationist policies and neutrality. The Neutrality Acts of 1935 and 1936 prohibited trade with any belligerents or loans to them. In 1937 the President was empowered to declare an arms embargo(禁运)in wars between nations at his discretion.

American opinion began to change somewhat after president Roosevelt’s “quarantine the aggressor” speech at Chicago (1937) in which he severely criticized Hitler’s policies. Germany’s seizure of Austria and the Munich Pact for the partition of Czechoslovakia (1938) also aroused the American people. The conquest of Czechoslovakia in March 1939 was another rude awakening to the menace of the Third Reich. In August 1939 came the shock of Nazi-Soviet Pact and in September the attack on Poland, the outbreak of European war. The United States attempted to maintain neutrality in spite of sympathy for the democracies arrayed against the Third Reich. The Neutrality Act of 1939 repealed the arms embargo and permitted “cash and carry” exports of arms to belligerent nations. A strong national defense program was begun. A draft act was passed (1940) to strengthen the military service. A Lend Lease Act (1941) authorized the President to sell, exchange, or lend materials to any country deemed necessary by him for the defense of the United States. Help was given to Britain by exchanging certain overage destroyers for the right to establish American bases in British territory in the Western Hemisphere. In August 1941, President Roosevelt and Prime Minister Churchill met and issued the Atlantic Charter that proclaimed the kind of a world which should be established after the war. In December 1941, Japan launched the unprovoked attack on the United States at Pearl Harbor. Immediately thereafter, Germany declared war on the United States.

1.One item occurring before 1937 that the author does not mention in his list of actions that alienated the American public was ___.
A.Nazi barbarism
B.The pacts with Italy
C.German plans for conquest
D.The burning of the Reichstag

2.The Neutrality Act of 1939 ___.
A.restated America’s isolationist policies
B.proclaimed American neutrality
C.permitted the selling of arms to belligerent nations
D.was a cause of our entrance into World War Ⅱ

3.An event that did not occur in 1939 was the ___.
A.invasion of Poland
B.invasion of Czechoslovakia
C.passing of the Neutrality Act
D.establishment of the University of Leipzig in Germany

4.The Lend Lease Act was blueprinted to ___.
A.strengthen our national defense
B.provide battleships to the Allies
C.help the British
D.promote the Atlantic Charter

5.The Neutrality Act of 1939 favored Great Britain because ___.
A.the British had command of the sea
B.the law permitted us to trade only with the Allies
C.it antagonized Japan
D.it led to the Lend Lease Act

第60篇答案:DCDAA

TOP

王长喜-六级考试标准阅读60篇(41-50)

第四十一篇:(Unit 11,Passage 1)

In the last 12 years total employment in the United States grew faster than at any time in the peacetime history of any country – from 82 to 110 million between 1973 and 1985 – that is, by a full one third. The entire growth, however, was in manufacturing, and especially in no – blue-collar jobs…

This trend is the same in all developed countries, and is, indeed, even more pronounced in Japan. It is therefore highly probable that in 25 years developed countries such as the United States and Japan will employ no larger a proportion of the labor force I n manufacturing than developed countries now employ in farming – at most, 10 percent. Today the United States employs around 18 million people in blue-collar jobs in manufacturing industries. By 2010, the number is likely to be no more than 12 million. In some major industries the drop will be even sharper. It is quite unrealistic, for instance, to expect that the American automobile industry will employ more than one –third of its present blue-collar force 25 years hence, even though production might be 50 percent higher.

If a company, an industry or a country does not in the next quarter century sharply increase manufacturing production and at the same time sharply reduce the blue-collar work force, it cannot hope to remain competitive – or even to remain “developed.” The attempt to preserve such blue – collar jobs is actually a prescription for unemployment…

This is not a conclusion that American politicians, labor leaders or indeed the general public can easily understand or accept. What confuses the issue even more it that the United States is experiencing several separate and different shifts in the manufacturing economy. One is the acceleration of the substitution of knowledge and capital for manual labor. Where we spoke of mechanization a few decades ago, we now speak of “robotization “ or “automation.” This is actually more a change in terminology than a change in reality. When Henry Ford introduced the assembly line in 1909, he cut the number of man – hours required to produce a motor car by some 80 percent in two or three years –far more than anyone expects to result from even the most complete robotization. But there is no doubt that we are facing a new, sharp acceleration in the replacement of manual workers by machines –that is, by the products of knowledge.

1.According to the author, the shrinkage in the manufacturing labor force demonstrates______.

A.the degree to which a country’s production is robotized
B.a reduction in a country’s manufacturing industries
C.a worsening relationship between labor and management
D.the difference between a developed country and a developing country

2.According to the author, in coming 25years, a developed country or industry, in order t remain competitive, ought to ______.

A.reduce the percentage of the blue-collar work force
B.preserve blue – collar jobs for international competition
C.accelerate motor – can manufacturing in Henry Ford’s style
D.solve the problem of unemployment

3.American politicians and labor leaders tend to dislike_____.

A.confusion in manufacturing economy
B.an increase in blue – collar work force
C.internal competition in manufacturing production
D.a drop in the blue – collar job opportunities

4.The word “prescription” in “a prescription for unemployment” may be the equivalent to ______

A.something recommended as medical treatment
B.a way suggested to overcome some difficulty
C.some measures taken in advance
D.a device to dire

5.This passage may have been excepted from ________

A.a magazine about capital investment
B.an article on automation
C.a motor-car magazine
D.an article on global economy

第41篇答案:AADCD


第42篇:(Unit 11,Passage 2)

What does the future hold for the problem of housing? A good deal depends, of course, on the meaning of “future”. If one is thinking in terms of science fiction and the space age, it is at least possible to assume that man will have solved such trivial and earthly problems as housing. Writers of science fiction, from H.G. Wells onwards, have had little to say on the subject. They have conveyed the suggestion that men will live in great comfort, with every conceivable apparatus to make life smooth, healthy and easy, if not happy. But they have not said what his house will be made of. Perhaps some new building material, as yet unimagined, will have been discovered or invented at least. One may be certain that bricks and mortar(泥灰,灰浆) will long have gone out of fashion.

But the problems of the next generation or two can more readily be imagined. Scientists have already pointed out that unless something is done either to restrict the world’s rapid growth in population or to discover and develop new sources of food (or both), millions of people will be dying of starvation or at the best suffering from underfeeding before this century is out. But nobody has yet worked out any plan for housing these growing populations. Admittedly the worst situations will occur in the hottest parts of the world, where housing can be light structure or in backward areas where standards are traditionally low. But even the minimum shelter requires materials of some kind and in the teeming, bulging towns the low-standard “housing” of flattened petrol cans and dirty canvas is far more wasteful of ground space than can be tolerated.

Since the war, Hong Kong has suffered the kind of crisis which is likely to arise in many other places during the next generation. Literally millions of refugees arrived to swell the already growing population and emergency steps had to be taken rapidly to prevent squalor(肮脏)and disease and the spread crime. The city is tackling the situation energetically and enormous blocks of tenements(贫民住宅)are rising at an astonishing aped. But Hong Kong is only one small part of what will certainly become a vast problem and not merely a housing problem, because when population grows at this rate there are accompanying problems of education, transport, hospital services, drainage, water supply and so on. Not every area may give the same resources as Hong Kong to draw upon and the search for quicker and cheaper methods of construction must never cease.

1.What is the author’s opinion of housing problems in the first paragraph?
A.They may be completely solved at sometime in the future.
B.They are unimportant and easily dealt with.
C.They will not be solved until a new building material has been discovered.
D.They have been dealt with in specific detail in books describing the future.

2.The writer is sure that in the distant future ___.

A.bricks and mortar will be replaced by some other building material.
B.a new building material will have been invented.
C.bricks and mortar will not be used by people who want their house to be fashionable.
D.a new way of using bricks and mortar will have been discovered.

3.The writer believes that the biggest problem likely to confront the world before the end of the century ___.

A.is difficult to foresee.
B.will be how to feed the ever growing population.
C.will be how to provide enough houses in the hottest parts of the world.
D.is the question of finding enough ground space.

4.When the writer says that the worst situations will occur in the hottest parts of the world or in backward areas, he is referring to the fact that in these parts ___.

A.standards of building are low.
B.only minimum shelter will be possible.
C.there is not enough ground space.
D.the population growth will be the greatest.

5.Which of the following sentences best summarizes Paragraph 3?

A.Hong Kong has faced a serious crisis caused by millions of refugees.
B.Hong Kong has successfully dealt with the emergency caused by millions of refugees.
C.Hong Kong’s crisis was not only a matter of housing but included a number of other problems of population growth.
D.Many parts of the world may have to face the kind of problems encountered by Hong Kong and may find it much harder to deal with them.

第四十二篇答案:AABDD

TOP

王长喜-六级考试标准阅读60篇(41-50)

第四十三篇:(Unit 11, Passage 3)

It is a curious paradox that we think of the physical sciences as “hard”, the social sciences as “soft,” and the biological sciences as somewhere in between. This is interpreted to mean that our knowledge of physical system is more certain than our knowledge of biological systems, and these in turn are more certain than our knowledge of social systems. In terms of our capacity of sample the relevant universes, however, and the probability that our images of these universes are at least approximately correct, one suspects that a reverse order is more reasonable. We are able to sample earth’s social systems with some degree of confidence that we have a reasonable sample of the total universe being investigated. Our knowledge of social systems, therefore, while it is in many ways extremely inaccurate, is not likely to be seriously overturned by new discoveries. Even the folk knowledge in social systems on which ordinary life is based in earning, spending, organizing, marrying, taking part in political activities, fighting and so on, is not very dissimilar from the more sophisticated images of the social system derived form the social sciences, even though it is built upon the very imperfect samples of personal experience.

In contrast, our image of the astronomical universe, or even if earth’s geological history, ca easily be subject to revolutionary changes as new data come in and new theories are worked out. If we define the “security” of our image of various parts of the total system as the probability of their suffering significant changes, then we would reverse the order for hardness and as the most secure, the physical sciences as the least secure, and again the biological sciences as somewhere in between. Our image of the astronomical universe is the least secure of all simply because we observe such a fantastically small sample of it and its record-keeping is trivial records of biological systems. Records of the astronomical universe, despite the fact that we learnt things as they were long age, are limited in the extreme.
Even in regard to such a close neighbor as the moon, which we have actually visited, theories about its origin and history are extremely different, contradictory, and hard to choose among. Our knowledge of physical evolution is incomplete and insecure.

1.The word “paradox” (Line 1, Para. 1) means “_____”.

A.implication            B.contradiction
C.interpretation         D.confusion

2.Accroding to the author, we should reverse our classification of the physical sciences as “hard” and the social sciences as “soft” because _______.

A.a reverse ordering will help promote the development of the physical sciences
B.our knowledge of physical systems is more reliable than that of social systems
C.our understanding of the social systems is approximately correct
D.we are better able to investigate social phenomena than physical phenomena

3.The author believes that our knowledge of social systems is more secure than that of physical systems because______.

A.it is not based on personal experience
B.new discoveries are less likely to occur in social sciences
C.it is based on a fairly representative quantity of data
D.the records of social systems are more reliable

4.The chances of the physical sciences being subject to great changes are the biggest because _____.

A.contradictory theories keep emerging all the time
B.new information is constantly coming in
C.the direction of their development is difficult to predict
D.our knowledge of the physical world is inaccurate

5.We know less about the astronomical universe than we don about any social system because ______.

A.theories of its origin and history are varied
B.our knowledge of it is highly insecure
C.only a very small sample of it has been observed
D.few scientists are involved in the study of astronomy

第43篇答案:ACDAD


第44篇:(Unit 11, Passage 4)

The promise of finding long-term technological solutions to the problem of world food shortages seems difficult to fulfill. Many innovations that were once heavily supported and publicized have since fallen by the wayside. The proposals themselves were technically feasible, but they proved to be economically unviable and to yield food products culturally unacceptable to their consumers.

One characteristic common to unsuccessful food innovations has been that, even with extensive government support, they often have not been technologically adapted or culturally acceptable to the people for whom they had been developed. A successful new technology, therefore, must fit the entire social cultural system in which it is to find a place. Security of crop yield, practicality of storage, and costs are much more significant than previously been realized by the advocates of new technologies.

The adoption of new food technologies depends on more than these technical and cultural considerations; economic factors and governmental policies also strongly influence the ultimate success of any innovation. Economists in the Anglo-American tradition have taken the lead in investigating the economics of technological innovation. Although they exaggerate in claiming that profitability is the key factor guiding technical change—they completely disregard the substantial effects of culture—they are correct in stressing the importance of profits. Most technological innovations in agriculture can be fully used only by large landowners and are only adopted if these profit-oriented business people believe that the innovation will increase their incomes. Thus, innovations that carry high rewards for big agribusiness groups will be adopted even if they harm segments of the population and reduce the availability of food in a country. Further, should a new technology promise to alter substantially the profits and losses associated with any production system, those with economic power will strive to maintain and improve their own positions. Therefore, although technical advances in food production and processing will perhaps be needed to ensure food availability, meeting food needs will depend much more on equalizing economic power among the various segments of the populations within the developing countries themselves.

1.The passage mentions all of the following as factors important to the success of a new food crop except the ___.

A.practicality of storage of the crop.
B.security of the crop yield.
C.quality of the crop’s protein.
D.cultural acceptability of the crop.

2.The author suggests that, in most emerging countries, extensive government intervention accompanying the introduction of a food innovation will ___.

A.usually be sufficient to guarantee the financial success of the innovation.
B.be necessary to ensure that the benefits of the innovation will be spread throughout the society.
C.normally occur only when the innovation favors large landowners.
D.generally cost the country more than will be earned by the innovation.

3.The first paragraph of the passage best supports which of the following statements?

A.Too much publicity can harm the chances for the success of a new food innovation.
B.Innovations that produce culturally acceptable crops will generally be successful.
C.A food-product innovation can be technically feasible and still not be economically viable.
D.It is difficult to decide whether a food-product innovation has actually been a success.

4.The author provides a sustained argument to uphold which of the following assertions?

A.Profitability is neither necessary nor sufficient for a new technology to be adopted.
B.Profitability is the key factor guiding technological change.
C.Economic factors and governmental policies strongly influence the ultimate success of any innovation.
D.Innovations carrying high rewards for big agribusiness groups harm the poor.

5.The primary purpose of the passage is to discuss the ___.

A.means of assessing the extent of the world food shortage.
B.difficulties of applying technological solutions to the problem of food shortages.
C.costs of introducing a new food technology into a developing country.
D.nature of the new technological innovations in the area of food production.

第四十四篇答案:CBCCB

TOP

王长喜-六级考试标准阅读60篇(41-50)

第四十五篇:(Unit 12,Passage 1)

What most people don’t realize is that wealth isn’t the same as income. If you make $ 1 million a year and spend $ 1 million, you’re not getting wealthier, you’re just living high. Wealth is what you accumulate, not what you spend.

The most successful accumulators of wealth spend far less than they can afford on houses, cars, vacations and entertainment. Why? Because these things offer little or no return. The wealthy would rather put their money into investments or their businesses. It’s an attitude.
Millionaires understand that when you buy a luxury house, you buy a luxury life –style too. Your property taxes skyrocket, along with the cost of utilities and insurance, and the prices of nearby services, such as grocery stores, tend to be higher.

The rich man’s attitude can also be seen in his car. Many drive old unpretentious sedans. Sam Walton, billionaire founder of the Wal – Mart Store, Inc., drove a pickup truck.
Most millionaires measure success by net worth, not income. Instead of taking their money home, they plow as much as they can into their businesses, stock portfolios and other assets. Why? Because the government doesn’t tax wealth; it taxes income you bring home for consumption, the more the government taxes.

The person who piles up net worth fastest tends to put every dollar he can into investments, not consumption. All the while, of course, he’s reinvesting his earnings from investments and watching his net worth soar. That’s the attitude as well.

The best wealth-builders pay careful attention to their money and seek professional advice. Those who spend heavily on cars, boats and buses, I’ve found, tend to skimp on investment advice. Those who skimp on the luxuries are usually more willing to pay top dollar for good legal and financial advice.

The self-made rich develop clear goals for their money. They may wish to retire early, or they may want to leave an estate to their children. The goals vary, but two things are consistent: they have   a dollar figure in mind-the amount they want to save by age 50, perhaps – and they work unceasingly toward that goal.

One thing may surprise you. If you make wealth – not just income – your goal, the luxury house you’ve been dreaming about won’t seem so alluring. You’ll have the attitude.

1.Which of the following statements is true?

A.Wealth is judged according to the life style one has.
B.Inheritance builds an important part in one’s wealth.
C.High income may make one live high and get rich t the same time.
D.Wealth is more of what one has made than anything else.

2.By the author’s opinion, those who spend money on luxury houses and cars_____.

A.will not be taxed by the government
B.have accumulated wealth in another sense
C.live high and have little saved
D.can show that they are among the rich

3.The rich put their money into business because_____.

A.they can get much in return to build their wealth
B.they are not interested in luxury houses and cars
C.their goal is to develop their company
D.that is the only way to spend money yet not to be taxed by the government

4.The U.S. government doesn’t tax what you spend money on _____.

A.cars    Bhouses     C.stock      D.boats

5.To become wealthy, one should______.

A.seek as much income as he can
B.work hard unceasingly
C.stick to the way he lives
D.save up his earnings

第45篇答案:DCACB


第46篇:(Unit 12,Passage 2)

It being not only possible but even easy to predict which ten-year-old boys are at greatest risk of growing up to be persistent offenders, what are we doing with the information? Just about the last thing that we should do is to wait until their troubles have escalated in adolescence and then attack them with the provisions of the new Criminal Justice Bill.

If this bill becomes law, magistrates will have the power to impose residential care orders. More young people will be drawn into institutional life when all the evidence shows that this worsens rather than improves their prospects. The introduction of short sharp shocks in detention centers will simply give more young people a taste of something else they don’t need; the whole regime of detention centers is one of toughening delinquents, and if you want to train someone to be anti-establishment, “I can’t think of a better way to do it,” says the writer of this report.

The Cambridge Institute of Criminology comes up with five key factors that are likely to make for delinquency: a low income family a large family, parents deemed by social workers to be bad at raising children, parents who themselves have a criminal record, and low intelligence in the child. Not surprisingly, the factors tend to overlap. Of the 63 boys in the sample who had at least three of them when they were ten, half became juvenile delinquents—compared with only a fifth of the sample as a whole.

Three more factors make the prediction more accurate: being judged troublesome by teachers at the age of ten, having a father with at least two criminal convictions and having another member of the family with a criminal record. Of the 35 men who had at least two of these factors in their background 18 became persistent delinquents and 8 more were in trouble with the law.

Among those key factors, far and away the most important was having a parent with a criminal record, even if that had been acquired in the distant past, even though very few parents did other than condemn delinquent behavior in their children.

The role of the schools emerges as extremely important. The most reliable prediction of all on the futures of boys came from teachers’ ratings of how troublesome they were at the age of ten. If the information is there in the classroom there must be a response that brings more attention to those troublesome children: a search for things to give them credit for other than academic achievement, a refusal to allow them to go on playing truant, and a fostering of ambition and opportunity which should start early in their school careers.

1.According to the author, delinquency should be tackled ___.

A.before adolescence
B.during institutional treatment
C.during adolescence
D.when the problem becomes acute

2.The number of young offenders could be reduced by the way of ___.

A.new legal measures
B.better residential care
C.brief periods of harsh punishment
D.examination of their backgrounds

3.What is the outcome result of putting young offenders into detention centers?

A.They become more violent
B.They receive useful training
C.They become used to institutions
D.They turn against society

4.Ten-year-old children likely to become offenders are usually___.

A.spoilt children from small families.
B.bright children in a poor family.
C.dull children with many brothers and sisters.
D.children whose parents have acquired wealth dishonestly.

5.The writer concludes that potential offenders could be helped by ___.

A.spending more time at school
B.more encouragement at school
C.more activities outside school
D.stricter treatment from teachers

第46篇答案:ADDCB

TOP

王长喜-六级考试标准阅读60篇(41-50)

第47篇:(Unit 12,Passage 3)

Personality is to large extent inherent. A-type parents usually bring A-type offspring. But the environment must also have a profound effect, since if competition is important to the parents, it is likely to become a major factor in the lives of their children.

One place where children soak up A characteristics is school, which is, by its very nature, a highly competitive institution. Too many schools adopt the “win at all costs” moral standard and measure their success by sporting achievements. The current passion for making children compete against their classmates or against the clock produces a two-layer system, in which competitive A types seem in some way better than their type B fellows. Being too keen to win can have dangerous consequences: remember that Pheidippides, the first marathon runner, dropped dead seconds after saying: “Rejoice, we conquer!”

By far the worst form of competition in schools is the disproportionate emphasis on examinations. It is a rare school that allows pupils to concentrate on those things they do well. The merits of competition by examination are somewhat questionable, but competition in the certain knowledge of failure is positively harmful.

Obviously, it is neither practical nor desirable that all A youngsters change into B’s. The world needs types, and schools have an important duty to try to fit a child’s personality to hide possible future employment. It is top management.

If the preoccupation of schools with academic work was lessened, more time might be spent teaching children surer values. Perhaps selection for the caring professions, especially medicine, could be made less by good grades in chemistry and more by such considerations as sensitivity and sympathy. It is surely a mistake to choose our doctors exclusively from A type stock. B’s are important and should be encouraged.

1.In Paragraph 2, Line 2, the word “institution” refers to ___.

A.establishment
B.social custom
C.law
D.school

2.According to the passage, A-type individuals are in most cases ___.

A.impatient
B.considerate
C.aggressive
D.agreeable

3.The author strongly objects to the practice of examination at schools because ___.

A.the pressure is too great on the students.
B.some students are bound to fail.
C.failure rates are too high.
D.the results of examinations are doubtful.

4.The selection of medical professionals is currently based on ___.

A.candidates’ sensitivity
B.academic achievements
C.competitive spirit
D.surer values

5.From the passage we can draw the conclusion that ___.

A.the personality of a child is well established at birth.
B.family influence dominates the shaping of one’s characteristics.
C.the development of one’s personality is due to multiple factors,
D.B-type characteristics can find no place in a competitive society.

第47篇答案:DCBBC


第48篇:(Unit 12,Passage 4)

The word religion is derived from the Latin noun religio, which denotes both earnest observance of ritual obligations and an inward spirit of reverence. In modern usage, religion covers a wide spectrum of meaning that reflects the enormous variety of ways the term can be interpreted. At one extreme, many committed believers recognize only their own tradition as a religion, understanding expressions such as worship and prayer to refer exclusively to the practices of their tradition. Although many believers stop short of claiming an exclusive status for their tradition, they may nevertheless use vague or idealizing terms in defining religion for example, true love of God, or the path of enlightenment. At the other extreme, religion may be equated with ignorance, fanaticism, or wishful thinking.

By defining religion as a sacred engagement with what is taken to be a spiritual reality, it is possible to consider the importance of religion in human life without making claims about what it really is or ought to be. Religion is not an object with a single, fixed meaning, or even a zone with clear boundaries. It is an aspect of human experience that may intersect, incorporate, or transcend other aspects of life and society. Such a definition avoid the drawbacks of limiting the investigation of religion to Western or biblical categories such as monotheism (belief in one god only) or to church structure, which are not universal. For example, in tribal societies, religion unlike the Christian church usually is not a separate institution but pervades the whole of public and private life.

In Buddhism, gods are not as central as the idea of a Buddha. In many traditional cultures, the idea of a sacred cosmic order is the most prominent religious belief. Because of this variety, some scholars prefer to use a general term such as the sacred to designate the common foundation of religious life.

Religion in this understanding includes a complex of activities that cannot be reduced to any single aspect of human experience. It is a part of individual life but also of group dynamics. Religion includes patterns of behavior but also patterns of language and thought. It is sometimes a highly organized institution that sets itself apart from a culture, and it is sometimes an integral part of a culture. Religious experience may be expressed in visual symbols, dance and performance, elaborate philosophical systems, legendary and imaginative stories, formal ceremonies, and detailed rules of ethical conduct and law. Each of these elements assumes innumerable cultural forms. In some ways there are as many forms of religious expression as there are human cultural environments.

1.What is the passage mainly concerned about?

A.Religion has a variety of interpretation.
B.Religion is a reflection of ignorance.
C.Religion is not only confined to the Christian categories.
D.Religion includes all kinds of activities.

2.What does the word “observance” probably convey in Para. 1?

A.notice
B.watching
C.conformity
D.experience

3.According to the passage what people generally consider religion to be?

A.Fantastic observance
B.Spiritual practice
C.Individual observance of tradition
D.A complex of activities

4.Which of the following is not true?

A.It is believed by some that religion should be what it ought to be.
B.“The path of enlightenment” is a definition that the author doesn’t agree to.
C.According to the author, the committed believers define religion improperly.
D.The author doesn’t speak in favor of the definition of “the sacred”.

5.Which of the following is religion according to the passage?

A.Performance of human beings.
B.Buddha, monotheism and some tribal tradition.
C.Practice separated from culture.
D.All the above.

第48篇答案:ACBDB

TOP

王长喜-六级考试标准阅读60篇(41-50)

第49篇:(Unit 13,Passage 1)

You stare at waterfall for a minute or two, and then shift your gaze to its surroundings. What you now see appears to drift upward.

These optical illusions occur because the brain is constantly matching its model of reality to signals from the body’s sensors and interpreting what must be happening—that your brain must have moved, not the other; that downward motions is now normal, so a change from it must now be perceived as upward motion.

The sensors that make this magic are of two kinds. Each eye contains about 120 million rods, which provide somewhat blurry black and white vision. These are the windows of night vision; once adapted to the dark, they can detect a candle burning ten miles away.

Color vision in each eye comes from six to seven million structures called cones. Under ideal conditions, every cone can “see” the entire rainbow spectrum of visible colors, but one type of cone is most sensitive to red, another to green, a third to blue.

Rods and cones send their messages pulsing an average 20 to 25 times per second along the optic nerve. We see an image for a fraction of a second longer than it actually appears. In movies, reels of still photographs are projected onto screens at 24 frames per second, tricking our eyes into seeing a continuous moving picture.

Like apparent motion, color vision is also subject to unusual effects. When day gives way to night, twilight brings what the poet T.S. Eliot called “the violet hour.” A light levels fall, the rods become progressively less responsive. Rods are most sensitive to the shorter wavelengths of blue and green, and they impart a strange vividness to the garden’s blue flowers.

However, look at a white shirt during the reddish light of sunset, and you’ll still see it in its “true” color—white, not red. Our eyes are constantly comparing an object against its surroundings. They therefore observe the effect of a shift in the color of illuminating on both, and adjust accordingly.

The eyes can distinguish several million graduations of light and shade of color. Each waking second they flash tens of millions of pieces of information to the brain, which weaves them incessantly into a picture of the world around us.

Yet all this is done at the back of each eye by a fabric of sensors, called the retina, about as wide and as thick as a postage stamp. As the Renaissance inventor and artist Leonardo da Vinci wrote in wonder, “Who would believe that so small a space could contain the images of all the universe?”

1.Visual illusions often take place when the image of reality is ___.

A.matched to six to seven million structures called cones.
B.confused in the body’s sensors of both rods and cones.
C.interpreted in the brain as what must be the case.
D.signaled by about 120 million rods in the eye.

2.The visual sensor that is capable of distinguishing shades of color is called ___.

A.cones
B.color vision
C.rods
D.spectrum

3.The retina send pulses to the brain ___.

A.in short wavelengths
B.as color pictures
C.by a ganglion cell
D.along the optic nerve.

4.Twenty-four still photographs are made into a continuous moving picture just because ___.

A.the image we see usually stays longer than it actually appears.
B.we see an object in comparison with its surroundings.
C.the eyes catch million pieces of information continuously.
D.rods and cones send messages 20 to 25 times a second.

5.The author’s purpose in writing the passage lies in ___.

A.showing that we sometimes are deceived by our own eyes.
B.informing us about the different functions of the eye organs.
C.regretting that we are too slow in the study of eyes.
D.marveling at the great work done by the retina.

第49篇答案:CADAB


第50篇:(Unit 13,Passage 2)

Art is considered by many people to be little more than a decorative means of giving pleasure. This is not always the case, however; at times, art may be seen to have a purely functional side as well. Such could be said of the sandpaintings of the Navaho Indians of the American Southwest; these have a medicinal as well as an artistic purpose.

According to Navaho traditions, one who suffers from either a mental or a physical illness has in come way disturbed or come in contact with the supernatural—perhaps a certain animal, a ghost, or the dead. To counteract this evil contact, the ill person or one of his relatives will employ a medicine man called a “singer” to perform a healing ceremony which will attract a powerful supernatural being.

During the ceremony, which may last from 2 to 9 days, the “singer” will produce a sandpainting on the floor of the Navaho hogan. On the last day of the ceremony, the patient will sit on this sandpainting and the “singer” will rub the ailing parts of the patient’s body with sand from a specific figure in the sandpainting. In this way the patient absorbs the power of that particular supernatural being and becomes strong like it. After the ceremony, the sandpainting is then destroyed and disposed of so its power will not harm anyone.

The art of sandpainting is handed down from old “singer” to their students. The material used are easily found in the areas the Navaho inhabit; brown, red, yellow, and white sandstone, which is pulverized by being crushed between 2 stones much as corns is ground into flour. The “singer” holds a small amount of this sand in his hand and lets it flow between his thumb and fore-finger onto a clean, flat surface on the floor. With a steady hand and great patience, he is thus able to create designs of stylized people, snakes and other creatures that have power in the Navaho belief system. The traditional Navaho does not allow reproduction of sandpaintings, since he believes the supernatural powers that taught him the craft have forbidden this; however, such reproductions can in fact be purchased today in tourist shops in Arizona and New Mexico. These are done by either Navaho Indians or by other people who wish to preserve this craft.

1.The purpose of the passage is to ___.

A.discuss the medical uses of sandpaintings in medieval Europe.
B.study the ways Navaho Indians handed down their painting art.
C.consider how Navaho “singer” treat their ailments with sandpaintings.
D.tell how Navaho Indians apply sandpainting for medical purposes.

2.The purpose of a healing ceremony lies in ___.

A.pleasing the ghosts
B.attracting supernatural powers
C.attracting the ghosts
D.creating a sandpainting

3.The “singer” rubs sand on the patient because ___.

A.the patient receives strength from the sand
B.it has pharmaceutical value
C.it decorates the patient
D.none of the above

4.What is used to produce a sandpainting?

A.Paint
B.Beach sand
C.Crushed sandstone
D.Flour

5.Which of the following titles will be best suit the passage?

A.A New Direction for Medical Research
B.The Navaho Indians’ Sandpainting
C.The Process of Sandpainting Creation
D.The Navaho Indians’ Medical History

第50篇答案:DBACB

TOP

王长喜-六级考试标准阅读60篇(31-40)

第31篇:(Unit 8, Passage 3)

For four lonely years, Evelyn Jones of Rockford, Illinois, lived friendless and forgotten in one room of a cheap hotel. “I wasn’t sick, but I was acting sick,” the 78-year-old widow says. “Every day was the same—I would just lie on my bed and maybe cook up some soup.” Then, six months ago, she was invited to “The Brighter Side”—Rockford’s day care center for the elderly. Every weekday morning since then, she has left her home to meet nine other old people in a church for a rich program of charity work, trips, games, and—most important of all—friendly companionship.

Just a few years ago, there were few choices for the elderly between a normal life in their own homes and being totally confined in nursing homes. Many of them were sent to rest homes long before they needed full-time care. Others like Mrs. Jones, were left to take care of themselves. But in 1971, the White House Conference on Aging called for the development of alternatives to care in nursing homes for old people, and since then, government-supported day-care programs like The Brighter Side have been developed in most big American cities.

“This represents a real alternative to the feared institution and makes old people believe they have not left the world of living,” says Alice Brophy, 64, director of New York City’s Office for the Aging. “They do well at the centers, and I hate it when people describe us as elderly playpens.” New York’s 138 centers encourage continuing contact for the aged with the community’s life. The centers serve more than 15,000 members, and volunteer workers are always looking for new ones. If someone doesn’t show up at the center for several days in a row, a worker at the center calls to make sure all is well. And although participation in the center is free, those who want to can pay for their lunches.

No normal studies have been made of these centers for the elderly, but government officials are enthusiastic. In the future, the Public Health Service will do a study to decide if the programs can receive federal Medicare money. And the old people themselves are very happy with the programs. “There is no way,” says Evelyn Jones, smiling at her new companions at the Brighter Side, “that I will ever go back to spending my day with all those loses at the hotel.”

1.What is the main idea of the article?

A.Day care centers may be able to receive federal Medicare money.
B.Day care centers can make life better for elderly people.
C.Many old people in the United States are lonely.
D.Old people have no place in their society.

2.According to Para 2, why did many old people have to go to nursing homes?

A.They need full-time care.
B.They wanted to go there.
C.They were sent there.
D.They were volunteers there.

3.According to Alice Brophy (in Paragraph 3)___.

A.the centers are like elderly playpens.
B.the old people do well at the day care centers.
C.old people like nursing institutions.
D.outside the Brighter side they don’t work for the old.

4.“This represents a real alternative to the feared institution.” (in Paragraph 3) In the sentence “this” means ___.

A.most big American cities.
B.rest homes.
C.day care programs.
D.the White House Conference on aging.

5.How does the writer of the article seem to feel about day care centers for the elderly?

A.The writer approves of them.
B.The writer disapproves of them.
C.The writer thinks nursing homes are better.
D.He doesn’t say anything about it.

第31篇答案:BCBCA

第32篇:(Unit 8, Passage 4)

Drunken driving--sometimes called America’s socially accepted form of murder--has become a national epidemic. Every hour of every day about three Americans on average are killed by drunken drivers, adding up to an incredible 250,000 over the past decade.

A drunken driver is usually defined as one with a 0.10 blood alcohol content or roughly three beers, glasses of wine or shots of whisky drunk within two hours. Heavy drinking used to be an acceptable part of the American macho image and judges were lenient in most courts, but the drunken slaughter has recently caused so many well-publicized tragedies, especially involving young children, that public opinion is no longer so tolerant.

Twenty states have raised the legal drinking age to 21, reversing a trend in the 1960s to reduce it to 18. After New Jersey lowered it to 18, the number of people killed by 18-20-year-old drivers more than doubled, so the state recently upped it back to 21.

Reformers, however, fear raising the drinking age will have little effect unless accompanied by educational programs to help young people to develop “responsible attitudes” about drinking and teach them to resist peer pressure to drink.

Though new laws have led to increased arrests and tests and, in many areas already, to a marked decline in fatalities. Some states are also penalizing bars for serving customers too many drinks. A tavern in Massachusetts was fined for serving six or more double brandies to a customer who “obviously intoxicated” and later drove off the road, killing a nine-year-old boy.

As the fatalities continue to occur daily in every state, some Americans are even beginning to speak well of the 13 years of national prohibition of alcohol that began in 1919, what President Hoover called the “ noble experiment.” They forgot that legal prohibition didn’t stop drinking, but encouraged political corruption and organized crime. As with the booming drug trade generally, there is no easy solution.

1.Drunken driving had become a major problem in America because ___.

A.most Americans are heavy drinkers.
B.Americans are now less shocked by road accidents.
C.accidents attract so much publicity.
D.drinking is a socially accepted habit in America.

2.Why has public opinion regarding drunken driving changed?

A.Because detailed statistics are now available.
B.Because the news media have highlighted the problem.
C.Because judges are giving more severe sentences.
D.Because drivers are more conscious of their image.

3.Statistics issued in New Jersey suggested that ___.

A.many drivers were not of legal age.
B.young drivers were often bad drivers.
C.the level of drinking increased in the 1960s.
D.the legal drinking age should be raised.

4.Laws recently introduced in some states have ___.

A.reduced the number of convictions.
B.resulted in fewer serious accidents.
C.prevented bars from serving drunken customers.
D.specified the amount drivers can drink.

5.Why is the problem of drinking and driving difficult to solve?

A.Because alcohol is easily obtained.
B.Because drinking is linked to organized crime.
C.Because legal prohibition has already failed.
D.Because legislation alone is not sufficient.

第三十二篇答案:DBDBD

TOP

王长喜-六级考试标准阅读60篇(31-40)

第三十三篇:(Unit 9,Passage 1)

Fresh water life itself, has never come easy in the Middle East. Ever since the Old Testament(旧约圣经) God punished man with 40 days and 40 nights of rain, water supplies here have been dwindling. The rainfall only comes in winter, Inshallah ----- Good willing –and drains quickly through the semiarid land, leaving the soil to bake and to thirst for next November.

The region’s accelerating population, expanding agriculture, industrialization, and higher living standards demand more fresh water. Drought and pollution limit its availability. War and mismanagement squander it. Says Joyce Starr of the Global Water Summit Initiative, based in Washington, D.C.” Nations like Israel and Jordan are swiftly sliding into that zone where they are suing all the water resources available to them. They have only 15 to 20 years left before their agriculture, and ultimately their food security, is threatened.”

I came here to examine this crisis in the making, to investigate fears that “water wars “are imminent, that water has replaced oil as the region’s most contentious commodity. For more than two months I traveled through three river valleys and seven nations -----from southern Turkey down the Euphrates River Syria, Iraq, and on to Kuwait; to Israel and Jordan, neighbors across the valley of the Jordan; to the timeless Egyptian Nile.

Even amid the scarcity there are haves and have – notes. Compared with the United States, which in 1990 had a freshwater potential of 10000 cubic meters(2.6 million galloons) a year for each citizen, Iraq had 5 500, Turkey had 4 000, and Syria had more than 2 800. Egypt’s potential was only 1 100. Israel had 460, Jordan a meager 260. But these are not firm figures, because upstream use of river water can dramatically alter the potential downstream.

Scarcity is only one element of the crisis. Inefficiency is another, as is the reluctance of some water – poor nations to change priorities from agriculture to less water – intensive enterprises. Some experts suggest that if nations would share both water technology and resources, they could satisfy the region’s population, currently 159 million. But in this patchwork of ethnic and religious rivalries, water seldom stands alone as an issue. It is entangled in the politics that keep people from trusting and seeking help from one another. Here, where water, like truth, is precious, each nation tends to find its own water and supply its own truth.

As Israeli hydrology professor Uri Shamir told me :” If there is political will for peace, water will not be a hindrance. If you want reasons to fight, water will not e a hindrance. If you want reasons to fight, water will give you ample opportunities.”

1.Why “for next November” (para.1)? Because________.

A.according to the Ole Testament fresh water is available only in November
B.rainfall comes only in winter starting form November
C.running water systems will not be ready until next November
D.it is a custom in that region that irrigation to crops is done only in November

2.What is the cause for the imminent water war?

A.Lack of water resources          B.Lack of rainfall
C.Inefficient use of water           D.All the above

3.One way for the region to use water efficiently is to _______

A.develop other enterprises that cost less water
B.draw a plan of irrigation for the various nations
C.import water from water – rich nations
D.stop wars of any sort for good and all

4.Uri Shamir’s viewpoint is that ________.

A.nations in that region are just fighting for water
B.people there are thirsty for peace instead of water
C.water is no problem as long as there is peace
D.those nations have every reason to fight for water

5.The author’s tone in the article can be described as ______-.

A.depressing     B.urgent      C.joking       D.mocking

第33篇答案:BDACB

第34篇:(Unit 9,Passage 2)

The British psychoanalyst John Bowlby maintains that separation from the parents during the sensitive “attachment” period from birth to three may scar a child’s personality and predispose to emotional problems in later life. Some people have drawn the conclusion from Bowlby’s work that children should not be subjected to day care before the age of three because of the parental separation it entails, and many people do believe this. But there are also arguments against such a strong conclusion.

Firstly, anthropologists point out that the insulated love affair between children and parents found in modern societies does not usually exist in traditional societies. For example, we saw earlier that among the Ngoni the father and mother of a child did not rear their infant alone--far from it. Secondly, common sense tells us that day care would not so widespread today if parents, caretakers found children had problems with it. Statistical studies of this kind have not yet been carried out, and even if they were, the results would be certain to be complicated and controversial. Thirdly, in the last decade, there have been a number of careful American studies of children in day care, and they have uniformly reported that day care had a neutral or slightly positive effect on children’s development. But tests that have had to be used to measure this development are not widely enough accepted to settle the issue.

But Bowlby’s analysis raises the possibility that early day care has delayed effects. The possibility that such care might lead to, say, more mental illness or crime 15 or 20 years later can only be explored by the use of statistics. Whatever the long-term effects, parents sometimes find the immediate effects difficult to deal with. Children under three are likely to protest at leaving their parents and show unhappiness. At the age of three or three and a half almost all children find the transition to nursery easy, and this is undoubtedly why more and more parents make use of child care at this time. The matter, then, is far from clear-cut, though experience and available evidence indicate that early care is reasonable for infants.

1.This passage primarily argues that ___.

A.infants under the age of three should not be sent to nursery schools.
B.whether children under the age of three should be sent to nursery schools.
C.there is not negative long-term effect on infants who are sent to school before they are three.
D.there is some negative effect on children when they are sent to school after the age of three.

2.The phrase “predispose to” (Para. 1, line 3) most probably means ___.

A.lead to
B.dispose to
C.get into
D.tend to suffer

3.According to Bowlby’s analysis, it is quite possible that ___.

A.children’s personalities will be changed to some extent through separation from their parents.
B.early day care can delay the occurrence of mental illness in children.
C.children will be exposed to many negative effects from early day care later on.
D.some long-term effects can hardly be reduced from children’s development.

4.It is implied but not stated in the second paragraph that ___.

A.traditional societies separate the child from the parent at an early age.
B.Children in modern societies cause more troubles than those in traditional societies.
C.A child did not live together with his parents among the Ngoni.
D.Children in some societies did not have emotional problems when separated from the parents.

5.The writer concludes that ___.

A.it is difficult to make clear what is the right age for nursery school.
B.It is not settled now whether early care is reasonable for children.
C.It is not beneficial for children to be sent to nursery school.
D.It is reasonable to subject a child above three to nursery school.

第三十四篇答案:BDCAD

TOP

王长喜-六级考试标准阅读60篇(31-40)

第三十五篇(Unit 9, Passage 3)

The life story of the human species goes back a million years, and there is no doubt that man came only recently to the western hemisphere. None of the thousands of sites of aboriginal (土著的) habitation uncovered in North and South America has antiquity comparable to that of old World sites. Man’s occupation of the New World may date several tens of thousands of years, but no one rationally argues that he has been here even 100,000 years.

Speculation as to how man found his way to America was lively at the outset, and the proposed routes boxed the compass. With one or two notable exceptions, however, students of American anthropology soon settled for the plausible idea that the first immigrants came b way of a land bridge that had connected the northeast comer of Asia to the northwest corner of North America across the Bering Strait. Mariners were able to supply the reassuring information that the strait is not only narrow – it is 56 miles wide – but also shallow, a lowering of the sea level there by 100 feet or so would transform the strait into an isthmus (地峡). With little eels in the way of evidence to sustain the Bering Strait land bridge, anthropologists (人类学家) embraced the idea that man walked dryshod (不湿鞋的) from Asia to America.

Toward the end of the last century, however, it became apparent that the Western Hemisphere was the New World not only for man but also for a host of animals and plants. Zoologists and botanists showed that numerous subjects of their respective kingdoms must have originated in Asia and spread to America. These findings were neither astonishing nor wholly unexpected. Such spread of populations is not to be envisioned as an exodus or mass migration, even in the case of animals. It is, rather, a spilling into new territory that accompanies increase in numbers, with movement in the direction of least population pressure and most favorable ecological conditions. But the immense traffic in plant and animal’s forms placed a heavy burden on the Bering Strait land bridge as the anthropologists ahead envisioned it. Whereas purposeful men could make their way across a narrow bridge, the slow diffusion of plant and animals would require an avenue as a continent and available for ages at a stretch.

1.The movement of plants and animals form Asia to America indicates ______.

A.that they could not have traveled across the Bering Strait
B.that Asia and the Western hemisphere were connected by a large land mass
C.that the Bering Sea was an isthmus at one time
D.that migration was in the one direction only

2.The author is refuting the notion that _____.

A.life arose in America independently of life in Europe
B.the first settlers in America came during the sixteenth century
C.a large continent once existed which has disappeared
D.man was a host to animals and plants

3.By using the words “boxed the compass “(in Line 7) the author implies that _____.

A.the migration of mankind was from West to East
B.the migration of mankind was from East to West
C.mankind traveled in all directions
D.mankind walked from Asia to America

4.One reason for the migration not mentioned by the author is _____.

A.overcrowding
B.favorable environmental conditions
C.famine
D.the existence of a land bridge

5.We may assume that in the paragraph that follows this passage the author argues about______.

A.the contributions of anthropologist
B.the contributions of zoologists and botanists
C.the contributions made by the American Indians
D.the existence of a large land mass between Asia and North America

第35篇答案:BCCCD

第36篇(Unit 9, Passage 4)

There was on shop in the town of Mufulira, which was notorious for its color bar. It was a drugstore. While Europeans were served at the counter, a long line of Africans queued at the window and often not only were kept waiting but, when their turn came to be served, were rudely treated by the shop assistants. One day I was determined to make a public protest against this kind of thing, and many of the schoolboys in my class followed me to the store and waited outside to see what would happen when I went in.

I simply went into the shop and asked the manager politely for some medicine. As soon as he saw me standing in the place where only European customers were allowed to stand he shouted at me in a bastard language that is only used by an employed when speaking to his servants. I stood at the counter and politely requested in English that I should be served. The manager became exasperated and said to me in English, “If you stand there till Christmas I will never serve you.”

I went to the District commissioner’s office. Fortunately the District Commissioner was out, for he was one of the old school; however, I saw a young District Officer who was a friend of mine. He was very concerned to hear my story and told me that if ever I wanted anything more from the drugstore all I had to do was come to him personally and he would buy my medicine for me. I protested that that was not good enough. I asked him to accompany me back to the store and to make a protest to the manager. This he did, and I well remember him saying to the manager, “Here is Mr. Kaunda who is a responsible member of the Urban Advisory Council, and you treat him like a common servant.” The manager of the drugstore apologized and said, “If only he had introduced himself and explained who he was, then, of course I should have given him proper service.”

I had to explain once again that he had missed my point. Why should I have to introduce myself every time I went into a store…any more than I should have to buy my medicine by going to a European friend? I want to prove that any man of any color, whatever his position, should have the right to go into any shop and buy what he wanted.

1.“Color bar” in the first paragraph comes closest in meaning to ___.

A.a bar which is painted in different colors.
B.the fact that white and black customers are served separately.
C.a bar of chocolate having different colors.
D.a counter where people of different colors are served with beer.

2.The writer was, at the time of the story, ___.

A.a black school teacher
B.an African servant
C.a black, but a friend of Europeans
D.a rich black

3.The manager of the drugstore shouted at the writer in a bastard language because ___.

A.he hadn’t learned to speak polite English.
B.he thought the writer wouldn’t understand English.
C.that was the usual language used by Europeans when speaking to Africans.
D.that was the only language he could speak when he was angry.

4.In the third paragraph, “he was one of the old school” means ___.

A.he believed in the age-old practice of racial discrimination.
B.he was a very old man.
C.he graduated from an old, conservative school.
D.he was in charge of an old school.

5.Why didn’t the writer wait at the window of the drugstore like other black African?

A.Because he thought he was educated and should be treated differently.
B.Because he thought, being an important person, he should not be kept waiting.
C.Because he thought his white friends would help him out.
D.Because he wanted to protest against racial discrimination.

第三十六篇答案:BACAD

TOP

王长喜-六级考试标准阅读60篇(31-40)

第三十七篇:(Unit 10,Passage 1)

Jogging has become the most popular individual sport in America. Many theories, even some mystical ones, have been advanced to explain the popularity of jogging. The plain truth is that jogging is a cheap, quick and efficient way to maintain (or achieve )physical fitness.
The most useful sort of exercise is exercise that develops the heart, lungs, and circulatory systems. If these systems are fit, the body is ready for almost any sport and for almost any sudden demand made by work or emergencies. One can train more specifically, as by developing strength for weight lifting or the ability to run straight ahead for short distances with great power s in football, but running trains your heart and lungs to deliver oxygen more efficiently to all parts of your body. It is worth noting that this sort of exercise is the only kind that can reduce heart disease, the number one cause of death in America.

Only one sort of equipment is needed – a good pair of shoes. Physicians advise beginning joggers not to run in a tennis or gym shoe. Many design advances have been made in only the last several years that make an excellent running shoe in dispensable if a runner wishes to develop as quickly as possible, with as little chance of injury as possible. A good running shoe will have a soft pad for absorbing shock, as well as a slightly built-up heel and a full heelcup that will give the knee and ankle more stability. A wise investment in good shoes will prevent bilisters and the foot, ankle and knee injures and will also enable the wearer to run on paved or soft surfaces.

No other special equipment is needed; you can jog in any clothing you desire, even your street clothes. Many joggers wear expensive, flashy warm –up suits, but just as many wear a simple pair of gym shoes and T-shirt; in fact, many people just jog in last year’s clothes. In cold weather, several layers of clothing are better than one heavy sweater or coat. If joggers are wearing several layers of clothing, they can add or subtract layers as conditions change.
It takes surprisingly little time to develop the ability to run. The American Jogging Association has a twelve – week program designed to move form a fifteen-minute walk (which almost anyone can manage who is in reasonable health) to a thirty-minute run. A measure of common sense, a physical examination, and a planned schedule are all it takes.

1.They main purpose of this passage is to _____.

A.discuss jogging as a physical fitness program
B.describe the type of clothing needed for jogging
C.provide scientific evidence of the benefits of jogging
D.distinguish between jogging as a “common sense “fitness program and a cult (崇拜) movement

2.The most useful kind of exercise is exercise that ______.

A.trains the body for weigh lifting
B.enables a person to run straight ahead for short distances with great power
C.is both beneficial and inexpensive
D.develops the heart, lungs, and circulatory systems

3.We can conclude from this passage that ______.

A.because of jogging, heart disease is no longer an American problem
B.jogging can be harmful if the runner is not properly prepared
C.warm-up suits are preferable to gym shoes and T-shirts
D.jogging is bad for the ankles and knees

4.The author’s tone ______.

A.skeptical        B.aggressive        C.approving        D.purely objective

5.As used in this passage, the word “mystical “ means ________.

A.awesome       B.horrifying         C.a spirtual discipline    D.vicious

第37篇答案:ADBCC

第38篇:(Unit 10,Passage 2)

There are spectacular differences between financial markets on the Continent of Europe on the one hand, and in Britain on the other hand. In Britain, the market is really the City of London. It is a free market, and it controls most of the flow of savings to investment. On the Continent, either a few banks or government officials direct the flow of funds to suit their economic plans. In Germany the flow is directed by all-powerful banks. In Britain there is more free interplay of market forces and far fewer regulations, rules and “red tape”. A French banker summed it up this way: “On the Continent you can’t do anything unless you’re been told you can; in England on the other hand you can do everything as long as you haven’t been told not to.”

There are many basic reasons for these differences. One is that Continental savers tend to prefer gold, cash or short-term assets. They invest only 10% of their savings in institutions like pension funds or insurance companies. But in Britain 50% of savings goes to them, and they, in turn, invest directly in equity market. A far lower proportion of savings is put in the banks in the form of liquid assets than on the Continent. Continental governments intervene directly or through the banks to collect savings together and transform them into medium or long-term loans for investment. The equity market is largely bypassed. On the Continent economic planning tends to be far more centralized than in Britain. In Britain it is possible to influence decisions affecting the country’s economy from within the City. It attracts a skilled and highly qualified work force. In France, on the other hand, an intelligent young man who wants a career in finance would probably find the civil service more attractive.

In Britain the market, or more accurately, money tends to be regarded as an end in itself. On the Continent it is regarded as a means to an end: investment in the economy. To British eyes continental systems with possible exception of the Dutch seem slow and inefficient. But there is one outstanding fact the City should not overlook. Britain’s growth rates and levels of investment over the last ten years have been much lower than on the Continent. There are many reasons for this, but the City must take part of the blame. If it is accepted that the basic function of a financial market is to supply industry and commerce with finance in order to achieve desired rates of growth, it can be said that by concentrating on the market for its own sake the City has tended to forget that basic function.

1.What is the best title of the passage?

A.Savings and the Growth Rate.
B.Banking and Finance: Two Different Realities.
C.Monetary Policy in Britain.
D.The European Continent and Britain.
2.What seems to be the most fundamental reason for this difference?

A.The British tend to regard money as an end, whereas Continental European consider it a means to an end.
B.The British invest only 10% of their savings in pension funds.
C.On the Continent you can’t do anything unless you have been told you can.
D.Intelligent young men who want a career tend to go to civil service on the Continent.

3.According to the passage, the Dutch way of finance and banking ___.

A.is similar to that of the French.
B.makes no difference whatever system it is compared to.
C.is perhaps resembling that of the British.
D.has a low efficiency.

4.The word “outstanding” in Line 4, Para 3___

A.beating
B.surplus
C.noticeable
D.seemingly

5.In what way does the continental system seem better?

A.The Continent maintains a higher growth rate and levels of investment.
B.It has less proportion of savings in the form of liquid assets.
C.It attracts intelligent young men.
D.In functions properly despite the fact that the British discount it.

第38篇答案:BACCA

TOP

王长喜-六级考试标准阅读60篇(31-40)

第39篇:(Unit 10, Passage 3)

The gift of being able to describe a face accurately is a rare one, as every experienced police officer knows to his cost. As the Lancet put it recently:” When we try to describe faces precisely words fail us, and we resort to identikit (拼脸型图) procedures.”

Yet, according to one authority on the subject, we can each probably recognise more than 1,000 faces, the majority of which differ in fine details. This, when one comes to think of it, is a tremendous feat, though, curiously enough, relatively little attention has been devoted to the fundamental problems of how and why we acquire this gift for recognizing and remembering faces. Is it an inborn property of our brains, or an acquired one? As so often happens, the experts tend to differ.

Thus, some argue that it is inborn, and that there are “special characteristics about the brain’s ability to distinguish faces”. In support of this these they note how much better we are at recognizing a face after a single encounter than we are, for example, in recognizing an individual horse. On the other hand, there are those, and they are probably in the majority, who claim that the gift is an acquired one.

The arguments in favour of this latter view, it must be confessed, are impressive. It is a habit that is acquired soon after birth. Watch, for instance, how a quite young baby recognises his member by sight. Granted that his other senses help – the sound other voice, his sense of smell, the distinctive way she handles him.

But of all these, sight is predominant. Formed at the very beginning of life, the ability to recognize faces quickly becomes an established habit, and one that is, essential for daily living, if not necessarily for survival. How essential and valuable it is we probably do not appreciate until we encounter people who have been deprived of the faculty.

This unfortunate inability to recognize familiar faces is known to all, but such people can often recognize individuals by their voices, their walking manners or their spectacles. With typical human ingenuity many of these unfortunate people overcome their handicap by recognizing other characteristic features.

1.It is stated in the passage that ______.

A.it is unusual for a person to be able to identify a face satisfactorily
B.the ability to recognize faces unhesitatingly is an unusual gift
C.quit a few people can visualize faces they have seen
D.few people can give exact details of the appearance of a face

2.What the author feels strange about is that _______.

A.people have the tremendous ability to recognize more than 1,000 faces
B.people don’t think much of the problem of how and why we acquire the ability to recognize and remember faces
C.people don’t realize how essential and valuable it is for them to have the ability to recognize faces
D.people have been arguing much over the way people recognize and remember faces

3.What is the first suggested explanation of the origin of the ability?

A.It is one of the characteristics peculiar to human beings.
B.It is acquired soon after birth.
C.It is something we can do from the very moment we are born.
D.It is learned from our environment and experiences.

4.According to the passage, how important is the ability to recognize faces?

A.It is useful in daily life but is not necessarily essential.
B.It is absence would make normal everyday life impossible.
C.Under certain circumstances we could not exist without it.
D.Normal social life would be difficult without it.

5.This passage seems to emphasize that ______.

A.the ability to recognize individuals is dependent on other senses as well as sight
B.sight is indispensable to recognizing individuals
C.the ability to recognise faces is a special inborn ability of the brain
D.the importance of the ability of recognize faces in fully appreciated by people.

第39篇答案:DBCBA

第40篇:(Unit 10, Passage 4)

Scattered through the seas of the world are billions of tons of small plants and animals called plankton. Most of these plants and animals are too small for the human eye to see. They drift about lazily with the currents, providing a basic food for many larger animals.

Plankton has been described as the equivalent of the grasses that grow on the dry land continents, and the comparison is an appropriate one. In potential food value however, plankton far outweighs that of the land grasses. One scientist has estimated that while grasses of the world produce about 49 billion tons of valuable carbohydrates each year. The sea’s plankton generates more than twice as much.

Despite its enormous food potential, little effort was made until recently to farm plankton as we farm grasses on land. Now marine scientists have at last begun to study this possibility, especially as the sea’s resources loom even more important as a means of feeding an expanding world population.

No one yet has seriously suggested that “planktonburgers” may soon become popular around the world. As a possible farmed supplementary food source, however, plankton is gaining considerable interest among marine scientists.

One type of plankton that seems to have great harvest possibilities is a tiny shrimplike creature called krill. Growing to two or three inches long, krill provide the major food for the giant blue whale, the largest animal ever to inhabit the Earth. Realizing that this whale may grow 100 feet and weigh 150 tons at maturity, it is not surprising that each one devours more than one ton of krill daily.

Krill swim about just below the surface in huge schools sometimes miles wide, mainly in the cold Antarctic. Because of their pink color, they often appear as a solid reddish mass when viewed from a ship or from the air. Krill are very high in food value. A pound of these crustaceans contains about 460 calories—about the same as shrimp or lobster, to which they are related.
If the krill can feed such huge creatures as whales, many scientists reason, they must certainly be contenders as new food source for humans.

1.Which of the following best portrays the organization of the passage?

A.The author presents the advantages and disadvantages of plankton as a food source.
B.The author quotes public opinion to support the argument for farming plankton.
C.The author classifies the different food sources according to amount of carbohydrate.
D.The author makes a general statement about plankton as a food source and then moves to a specific example.

2.According to the passage, why is plankton regarded to be more valuable than land grasses?

A.It is easier to cultivate.
B.It produces more carbohydrates.
C.It does not require soil.
D.It is more palatable.

3.Why does the author mention “planktonburgers”?

A.To describe the appearance of one type of plankton.
B.To illustrate how much plankton a whale consumes.
C.To suggest plankton as a possible food sources.
D.To compare the food values of beef and plankton.

4.What is mentioned as one conspicuous feature of krill?

A.They are the smallest marine animals.
B.They are pink in color.
C.They are similar in size to lobsters.
D.They have grass like bodies.

5.The author mentions all of the following as reasons why plankton could be considered a human food source except that it is ___.

A.high in food value.
B.in abundant supply in the oceans.
C.an appropriate food for other animals.
D.free of chemicals and pollutants.

第四十篇答案:DBCBD

TOP

王长喜-六级考试标准阅读60篇(21-30)

第21篇(Unit 6,Passage 1)

The Reader’s digest investigation asked Americans which was the biggest threat to the nation’s future—big business, big labor or big government. A whopping 67 percent replied “ big government”.

Opinion researchers rarely see such a vast change in public attitude. When put in historical perspective, from the time of Franklin Roosevelt’s New Deal to the present, the fallen status of government as a protector and benefactor is extraordinary. We’ve returned to the instinctive American wariness of Washington so common before the Great Depression.

In our poll, taken before the November elections, the overwhelming majority of our respondents wanted to stop or roll back the impact of government. In answer to another question posed by The Digest, 79 percent said they wanted either no more than the current level of government services and taxes, or less government and lower taxes.

“It seems to me that we in the middle class bear most of the burden,” says Jone Nell Norman, 61, a nurse in Dyersburg, Tenn., who often wonders about the government’s judgement in spending her money.

Of Americans in our sample, 62 percent believe that politician’s ethics and honesty have fallen. And what about Congress? Is it doing a good job? Or do members “ spend more time thinking about their political futures than passing good legislation?” Across generations, a thumping 89 percent thought the latter. “Congress always seems to be screwing up,” says one young Xer.

However, Americans are satisfied with their own lives and jobs. Four of five respondents were “completely “ or “ somewhat “ satisfied. The figures held up across all ages – including Xer, whom many pundits have claimed are pessimistic about their future.

Looking deeper at jobs, we found 70 percent of Americans believe they are about where they should be, given their talents and effort. This is an issue where age always makes a difference, since older people, who are more established in their jobs tend to be more satisfied, while younger workers are still trying to find the right niche. Sure enough, Xers scored 65 percent, about five points below average.

1.The U.S. government status in the public mind before the Great Depression ____.
A.was regarded as quite normal
B.used to be very low
C.remained a difficult problem for the federal government
D.reminded people of the principles laid down by Washington

2.”Xers” is repeated several times to refer to
  A.accusers                                       B.younger respondents
  C.college students                               D.blue-collar workers

3.The 61-years-old nurse Norman is mentioned in the article to show that ____.
A.the government has cheated her out of her money
B.it is hard for her to earn a living
C.even a retired nurse has lost faith in the government
D.the more the government does the greater stake tax – payers’ money will be put at

4.”Screwing up “ in paragraph 5 may be paraphrased as ____.
  A.indecisive in making decisions                 B.benefiting the nation in earnest
  C.making a mess of everything                    D.debating hotly

5.”Political future “ in paragraph 5 may be paraphrased as ____.
  A.the future of the whole nation                 B.people’s well – being in the future
  C.a position of higher rank                      D.awareness of consistency in policies

第21篇答案:BBDCC


第22篇(Unit 6,Passage 2)

Everyone has a moment in history, which belongs particularly to him. It is the moment when his emotions achieve their most powerful sway over him, and afterward when you say to this person “the world today” or “life” or “reality” he will assume that you mean this moment, even if it is fifty years past. The world, through his unleashed(释放的)emotions, imprinted itself upon him, and he carries the stamp of that passing moment forever.

For me, this moment—four years in a moment in history—was the war. The war was and is reality for me. I still instinctively live and think in its atmosphere. These are some of its characteristics: Franklin Delano Roosevelt is the president of the United States, and he always has been. The other two eternal world leaders are Winston Churchill and Joseph Stalin. America is not, never has been, and never will be what the song and poems call it, a land of plenty. Nylon, meat, gasoline, and steel are rare. There are too many jobs and not enough workers. Money is very easy to earn but rather hard to spend, because there isn’t very much to buy. Trains are always late and always crowded with “service men”. The war will always be fought very far from America, and it will never end. Nothing in America stands still for very long, including the people who are always either leaving or on leave. People in America cry often. Sixteen is the key and crucial and natural age for a human being to be, and people of all other ages are ranged in an orderly manner ahead of and behind you as a harmonious setting for the sixteen-year-olds of the world. When you are sixteen, adults are slightly impressed and almost intimidated by you. This is a puzzle finally solved by the realization that they foresee your military future: fighting for them. You do not foresee it. To waste anything in America is immoral. String and tinfoil are treasures. Newspapers are always crowed with strange maps and names of towns, and every few months the earth seems to lurch(突然倾斜)from its path when you see something in the newspapers, such as the time Mussolini, who almost seemed one of the eternal leaders, is photographed hanging upside down on a meat hook.

1.Which statement best depicts the main idea of the first paragraph?
A.Reality is what you make of it.
B.Time is like a river.
C.Emotions are powerful.
D.Every person has a special moment.

2.Why does the author still clearly remember the war?
A.Franklin Delano Roosevelt was President.
B.It was his personal reality and part of his life.
C.There was not much to buy.
D.The war would never end.

3.Which statement best describes the author’s feelings about the war?
A.It was ever real for him, yet he was not actively involved.
B.It was real for him because he was a soldier at that time.
C.It was very unreal to him.
D.The war was very disruptive to the people at home.

4.Why does the author think that adults are impressed with sixteen-year-olds?
A.Adults would like to be young.
B.Sixteen-year-olds do not waste things.
C.Sixteen-year-olds read newspapers.
D.They will be fighting soon for adults.

5.Why does the author say that string and tinfoil are treasures?
A.The war has made them scarce.
B.They are useful to sixteen-year-olds.
C.He liked them when he was sixteen.
D.People are very wasteful.

第二十二篇答案:DBADA

TOP

王长喜-六级考试标准阅读60篇(21-30)

第二十三篇:(Unit 6,Passage 3)

In general, our society is becoming one of giant enterprises directed by a bureaucratic(官僚主义的) management in which man becomes a small, well-oiled cog in the machinery. The oiling is done with higher wages, Nell-ventilated factories and piped music, and by psychologists and “human – relations” experts; yet all this oiling does not alter the fact that man has become powerless, that he is bored with it. In fact, the blue and the white-collar workers have become economic puppets who dance to the tune of automated machines and bureaucratic management.
The worker and employee are anxious, not only because they might find themselves out of a job; they are anxious also because they are unable to acquire any real satisfaction of interesting life. They live an die without ever having confronted the fundamental realities of human existence as emotionally and intellectually independent and productive human beings.

Those higher up on the social ladder are no less anxious. Their lives are no less empty than those of their subordinates. They are even more insecure in some respects. They are in a highly competitive race. To be promoted or to fall behind is not a matter of salary but even more a matter of self-respect. When they apply for their first job, they are tested for intelligence as well as for the right mixture of submissiveness and independence. From the moment on they are tested again and again – by the psychologists, for whom testing is a big business, and by their superiors, who judge their behavior, sociability, capacity to get along, etc. This constant need to prove that one is as good as or better than one’s fellow – competitor creates constant anxiety and stress, the very causes of unhappiness and illness.

Am I suggesting that we should return to the preidustrial mode of production or to nineteenth-century “free enterprise “ capitalism? Certainly not. Problems are never solved by returning to a stage which one has already outgrown. I suggest transforming our social system form a bureaucratically managed industrialism in which maximal production and consumption are ends in themselves into a humanist industrialism in which man and full development of his potentialities – those of all love and of reason – are the aims of social arrangements. Production and consumption should serve only as means to this end, and should be prevented from ruling man.

1.By “ a well-oiled cog in the machinery “ the author intends to deliver the idea that man is ____.
A.a necessary part of the society though each individual’s function is negligible
B.working in complete harmony with the rest of the society
C.an unimportant part in comparison with the rest of the society
D.a humble component of the society, especially when working smoothly

2.The real cause of the anxiety of the workers and employees is that ____.
A.they are likely to lose their hobs
B.they have no genuine satisfaction or interest in life
C.they are faced with the fundamental realities of human existence
D.they are deprived of their individuality and independence

3.From the passage we can conclude that real happiness of life belongs to those _____.
A.who are at the bottom of the society
B.who are higher up in their social status
C.who prove better than their fellow – competitors
D.who could dip far away from this competitive world

4.To solve the present social problems the author puts foruard a suggestion that we should ______.
A.resort to the production mode of our ancestors
B.offer higher wages to the workers and employees
C.enable man to fully develop his potentialities
D.take the fundamental realities for granted

5.The author’s attitude towards industrialism might best be summarized as one of ______.
A.approval                B.dissatisfaction
C.suspicion               D.susceptibility

第23篇答案:CDDCB


第24篇:(Unit 6,Passage 4)

Is it possible to persuade mankind to live without war? War is an ancient institution, which has existed for at least six thousand years. It was always bad and usually foolish, but in the past human race managed to live with it. Modern ingenuity has changed this. Either man will abolish war, or war will abolish man. For the present, it is nuclear weapons that cause the most serious danger, but bacteriological or chemical weapons may, before long, offer an even greater threat. If we succeed in abolishing nuclear weapons, our work will not be done. It will never be done until we have succeeded in abolishing war. To do this, we need to persuade mankind to look upon international questions in a new way, not as contests of force, in which the victory goes to the side which is most skillful in killing people, but by arbitration in accordance with agreed principles of law. It is not easy to change very old mental habits, but this is what must be attempted.

There are those who say that the adoption of this or that ideology would prevent war. I believe this to be a big error. All ideologies are based upon dogmatic statements that are, at best, doubtful, and at worst, totally false. Their adherents believe in them so fanatically that they are willing to go to war in support of them.

The movement of world opinion during the past few years has been very largely such as we can welcome. It has become a commonplace that nuclear war must be avoided. Of course very difficult problems remain in the world, but the spirit in which they are being approached is a better one than it was some years ago. It has begun to be thought, even by the powerful men who decide whether we shall live or die, that negotiations should reach agreements even if both sides do not find these agreements wholly satisfactory. It has begun to be understood that the important conflict nowadays is not between different countries, but between man and the atom bomb.

1.This passage implies that war is now ___.
A.worse than in the past.
B.as bad as in the past
C.not so dangerous as in the past
D.as necessary as in the past

2.In the sentence “To do this, we need to persuade mankind” (Para 1), “this” refers to ___.
A.abolish war
B.improve weapons
C.solve international problems
D.live a peaceful life

3.From Paragraph 2 we learn that the author of the passage ___.
A.is an adherent of some modern ideologies.
B.does not think that adoption of any ideology could prevent war.
C.believe that the adoption of some ideology could prevent war.
D.does not doubt the truth of any ideologies.

4.According to the author, ___.
A.war is the only way to solve international disputes.
B.war will be less dangerous because of the improvement of weapons.
C.it is impossible for the people to live without war.
D.war must be abolished if man wants to survive.

5.The last paragraph suggests that ___.
A.international agreements can be reached more easily now.
B.man begins to realize the danger of nuclear war.
C.nuclear war will definitely not take place.
D.world opinion welcomes nuclear war

第二十四篇答案:AABDB

TOP

王长喜-六级考试标准阅读60篇(21-30)

第二十五篇:(Unit 7, Passage 1)

Western airliner manufacturers seem to be tripping over themselves in their eagerness to sign collaborative agreements with Asian partners as a low-cost route to developing new airliners. Their potential Asian partners seem to be tripping over themselves to sign such agreements, as a low-cost route to acquiring new airliner technology. If they are not careful the two sides will end up tripping over each other: the one by selling its birth-right for short-term gain, the other by trying to break into a market which isn’t big enough to sustain it.

Technology transfer works in a growing market, where the aspirations of the new entrant receiving that technology can be met through expansion. The airliner market is not such a device.

Even the most optimistic projections of airliner sales for the next 20 years show that airliner manufacture can only be profitable if a small number of aircraft builders share the available sales. It follows that if new manufacturers come into the market and take sales, their sales must come from substitution, not expansion.

Given the complexity of today’s airliners, it is unlikely that any new entrant will have both the financial and technical resources to come into the market without the involvement of an established manufacturer. In the short term, such involvement may not be to the exclusive benefit of the new entrant: most of the established manufacturers are searching for ways to reduce costs of manufacture.

In the short term,, it can be of benefit to an established Western manufacturer to have either components of complete air – frames made or assembled in lower-wage economics such a China, Taiwan or Korea, while retaining the design, development and marketing of aircraft for itself. It would be a very unwise Western manufacturer which did not heed the fact that these developing economies are acquiring skills ( like computing ) at least as quickly as they are acquiring skills in metallbashing.

The danger comes when the new entrant no longer needs the established Western partner because it has acquired the technical and intellectual ability to design and build its own aircraft. An Asian partner may well find itself in the happy position of having the low-cost labour base, the high-cost technology base and the vital financial base to build a new airliner.

1.The author’s attitude towards Western/eastern collaboration can be depicted as ________.
A.positive        B.progressive         C.conservative        D.negative

2.”The airliner market is not such a device “ means that the airliner market _______.
A.does not encourage technology transfer
B.is too limited to offer chances of success
C.requires hi-tech rather than unaccepted devices
D.is full of competitions even for new entrants

3.Established manufacturers  search for partners in order to _______.
A.save the cost of the airframe         B.improve some aircraft components
C.save the cost of labour              D.develop new technology

4.According to the author, a wise established manufacturer should ______.
A.try to benefit from both financial and technical resources
B.break up his partnership with the East once profits are made
C:keep a tight told over hi-tech development and marketing of airliners
D.collaborate with Asian partners for a short time

5.The word “base” in the last paragraph represents_______.
A.a production place
B.the initial operation of building aircraft
C.a research institute
D.a position where to start building

第二十五篇答案:CDADA


第二十六篇:(Unit 7, Passage 2)

There are various ways in which individual economic units can interact with one another. Three basic ways may be described as the market system, the administered system, and the traditional system.

In a market system individual economic units are free to interact among each other in the marketplace. It is possible to buy commodities from other economic units or sell commodities to them. In a market, transactions may take place via barter or money exchange. In a barter economy, real goods such as automobiles, shoes, and pizzas are traded against each other. Obviously, finding somebody who wants to trade my old car in exchange for a sailboat may not always be an easy task. Hence, the introduction of money as a medium of exchange eases transactions considerably. In the modern market economy, goods and services are bought or sold for money.

An alternative to the market system is administrative control by some agency over all transactions. This agency will issue edicts or commands as to how much of each good and service should be produced, exchanged, and consumed by each economic unit. Central planning may be one way of administering such an economy. The central plan, drawn up by the government, shows the amounts of each commodity produced by the various firms and allocated to different households for consumption. This is an example of complete planning of production, consumption, and exchange for the whole economy.

In a traditional society, production and consumption patterns are governed by tradition; every person’s place within the economic system is fixed by parentage, religion, and custom. Transactions take place on the basis of tradition, too. People belonging to a certain group or caste may have an obligation to care for other persons, provide them with food and shelter, care for their health, and provide for their education. Clearly, in a system where every decision is made on the basis of tradition alone, progress may be difficult to achieve. A stagnant society may result.

1.What is the main purpose of the passage?
A.To outline contrasting types of economic systems.
B.To explain the science of economics.
C.To argue for the superiority of one economic system.
D.To compare barter and money-exchange markets.

2.In the second paragraph, the word “real” in “real goods” could best be replaced by ___,
A.high quality
B.concrete
C.utter
D.authentic.

3.According to the passage, a barter economy can generate ___.
A.rapid speed of transactions.
B.misunderstandings.
C.inflation
D.difficulties for the traders.

4.According to the passage, who has the greatest degree of control in the administered system?
A.Individual households
B.Small businesses.
C.Major corporations.
D.The government.

5.Which of the following is not mentioned by the author as a criterion for determining a person’s position in a traditional society?
A.Family background
B.Age
C.Religious beliefs.
D.Custom

第26篇答案:ABDDB

TOP

王长喜-六级考试标准阅读60篇(21-30)

第27篇:(Unit 7, Passage 3)

The American economic system is organized around a basically private-enterprise, market-oriented economy in which consumers largely determine what shall be produced by spending their money in the marketplace for those goods and services that they want most. Private businessmen, striving to make profits, produce these goods and services in competition with other businessmen; and the profit motive, operating under competitive pressures, largely determines how these goods and services are produced. Thus, in the American economic system it is the demand of individual consumers, coupled with the desire of businessmen to maximize profits and the desire of individuals to maximize their incomes, that together determine what shall be produced and how resources are used to produce it.

An important factor in a market-oriented economy is the mechanism by which consumer demands can be expressed and responded to by producers. In the American economy, this mechanism is provided by a price system, a process in which prices rise and fall in response to relative demands of consumers and supplies offered by seller-producers. If the product is in short supply relative to the demand, the price will be bid up and some consumers will be eliminated from the market. If, on the other hand, producing more of a commodity results in reducing its cost, this will tend to increase the supply offered by seller-producers, which in turn will lower the price and permit more consumers to buy the product. Thus, price is the regulating mechanism in the America economic system.

The important factor in a private-enterprise economy is that individual are allowed to own productive resources (private property), and they are permitted to hire labor, gain control over natural resources, and produce goods and services for sale at a profit. In the American economy, the concept of private property embraces not only the ownership of productive resources but also certain rights, including the right to determine the price of a product or to make a free contract with another private individual.

1.In Para. 1, “ the desire of individuals to maximize their incomes” means ___.
A.Americans never feel satisfied with their incomes.
B.Americans tend to overstate the amount of their incomes.
C.Americans want to have their incomes increased.
D.Americans want to increase the purchasing power of their incomes.

2.The first two sentences in the second paragraph clarity the idea to us that ___.
A.producers can satisfy the consumers by mechanized production.
B.consumers can express their demands through producers.
C.producers decide the prices of products.
D.supply and demand regulate prices.

3.The word “embraces” in Para. 3 probably parallels ___.
A.enfold
B.hug
C.comprehend
D.support

4.According to the passage, a private-enterprise economy is characterized by ___.
A.private property and rights concerned.
B.manpower and natural resources control.
C.ownership of productive resources
D.free contracts and prices.

5.The passage is mainly talking about ___.
A.how American goods are produced.
B.how American consumers buy their goods.
C.how American economic system works.
D.how American businessman make their profits.

第27篇答案:DDCAC


第28篇:(Unit 7, Passage 4)

The government-run command post in Tunis is staffed around the clock by military personnel, meteorologists and civilians. On the wall are maps, crisscrossed with brightly colors arrows that painstakingly track the fearsome path of the enemy.

What kind of invader gives rise to such high-level monitoring? Not man, not beast, but the lowly desert locust.(蝗虫) In recent months, billions of the 3-inch-long winged warriors have descended on Algeria, Libya, Morocco and Tunisia, blackening the sky and eating up crops and vegetation. The insect invasion, the worst in 30 years, is already creating great destruction in the Middle East and is now treating southern Europe. The current crisis began in late 1985 near the Red Sea. Unusually rainy weather moistened the sands of the Sudan, making them ideal breeding grounds for the locust, which lays its eggs in the earth. The insect onslaught threatens to create yet another African famine. Each locust can eat its weight (not quite a tenth of an ounce) in vegetation every 24 hours. A good-size swarm of 50 billion insects eats up 100,000 tons of grass, trees and crops in a single night.

All ﹩150 million may be needed this year. The U.S. has provided two spraying planes and about 50,000 gal. of pesticide. The European Community has donated ﹩3.8 million in aid and the Soviet Union, Canada, Japan and China have provided chemical-spraying aircraft to help wipe out the pests. But relief efforts are hampered by the relative mildness of approved pesticides, which quickly lose their deadly punch and require frequent replications. The most effective locust killer dieldrin has been linked to cancer and is banned by many Western countries and some of the affected African nations. More then 5 million acres have been dusted with locust-killing chemicals; another 5 million will be treated by the end of June.

On May 30, representatives of Tunisia, Algeria, Libya, Morocco and Mauritania will meet in Algiers to discuss tactics to wipe out the ravenous swarms. The move is an important step, but whatever plan is devised, the locust plague promised to get worse before the insects can be brought under control.

1.The main idea of the first sentence in the passage is that ___.
A.the command post is stationed with people all the time.
B.the command post is crowed with people all the time.
C.there are clocks around the command post.
D.the clock in the command post is taken care of by the staff.

2.The favorable breeding ground for the locust is ___.
A.rich soil.
B.wet land
C.paces covered crops and vegetation
D.the Red Sea

3.People are alert at the threat of the locust because ___.
A.the insects are likely to create another African famine.
B.the insects may blacked the sky.
C.the number of the insects increases drastically.
D.the insects are gathering and moving in great speed.

4.Which of the following is true?
A.Once the pesticides are used, locust will die immediately.
B.Relief efforts are proved most fruitful due to the effectiveness of certain pesticides.
C.Dieldrin, the most effective locust killer, has been widely accepted in many countries.
D.Over 10 million acres of affected area will have been treated with locust-killing chemicals by the end of June.

5.The purpose for affected nations to meet in Algiers on May 30 is ___.
A.to devise antilocust plans.
B.to wipe out the swarms in two years.
C.to call out for additional financial aid from other nations.
D.to bring the insects under control before the plague gets worse.

第二十八篇答案:BBADA

TOP

王长喜-六级考试标准阅读60篇(21-30)

第二十九篇:(Unit 8, Passage 1)

Improbable as it may seem, an increasing number of Germans are giving up their elegant Mercedeses, sleek BMWs and ferociously fast Porsches and getting behind the wheels of imported American models – fro plush Cadillacs to more prosaic Fords. Unlike the cars produced by Detroit’s European subsidiaries, these cars are as American as apple pie and watery beer. And thanks to a favorable exchange rate, they are more affordable than ever Last year Germans bought 12 477 new U.S. –built cars; sales are expected to double this year.

Like blue jeans, this buy – America fad appeals to Germans from all walks of life. Once regarded as faulty, flashy, gas – guzzling Goliaths, American autos are – thanks in large measure to foreign competition –more stylish and reliable than in years past. Tugged, off- road vehicles like the four-wheel drive Jeep Cherokee are now the hot wheels to drive among Germany’s thirty- something set. Owners and Aficionados of American – made care also boast their cars are cheaper to maintain.

But that’s not the main reason German motorists are choosing U.S. imports – It’s their price. Even after the cost of overseas shipping is included, American – made cars offer more value – and deluxe features – for less money than German models. A Chrysler LeBaron convertible sells for 35 000 marks; a BMW 320i convertible, by comparison, commands 10 000 marks more. And U.S. autos come with standard equipment – electric windows, automatic locks and sun roofs – that’s available only as expensive options on German models.

Owning an American car in Germany is not for everybody. But the worst headaches come form the German bureaucracy. Johann Erben, a Greiburg dental lab technician, purchased a LeBaron convertible during a U.S. trip in November – and has yet to drive it one kilometer. First, he waited months for the proper registration documents to arrive; then he spent more than 1 000 marks to have it comply with German regulations. Even so, safety inspectors refused to approve it until he changed the headlights and windows to European Community standards. “There I was with my supermodern, $ 20,000 car and unable to get it through inspection,” Erben recalled.

1.Detroit’s European subsidiaries _______.
A.produce the same models as Detroit supplies in the U.S. market
B.provide cars of European styles
C.produce cars that are thought to be un-American by Germans
D.could hardly meet the demand for American cars last year

2.The buy-American fad that appeals to Germans most seems to be _______.
A.blue jeans                B.apple pie
C.U.S.-made cars         D.watery beer

3.As for Germans, American cars not only are cheaper but _______.
A.endures wear and tear      B.are adaptable to road conditions
C.provides greater space      D.offers more deluxe features

4.Which of the following statements is true?
A.American cars used to consume a lot of oil.
B.Japanese cars still lead the German market.
C.The U.S. motor industry is now confident to cope with recession.
D.German cars are going to provide the same standard equipment as American-made cars.

5.European Community standards probably are _______.
A.a law to control the amount of imported goods from other continents
B.a set of standards to inspect imported cars
C.a system to regulate measures of manufactured goods
D.a set of standards to control product quality

第29篇答案:CADAC


第30篇:(Unit 8, Passage 2)

How often do you sit still and do absolutely nothing? The usual answer these days is “never”, or “hardly ever”. As the pace of life continues to increase, we are fast losing the art of relaxation. Once you are in the habit of rushing through life, being on the go from morning till night, it is hard to slow down and unwind. But relaxation is essential for a healthy mind and body.

Stress is a natural part of everyday life. There is no way to avoid it, since it takes many and varied forms--driving in traffic, problems with personal relationships are all different forms of stress. Stress, in fact, is not the “baddy” it is often reputed to be. A certain amount of stress is vital to provide motivation and give purpose to life. It is only when the stress gets out of control that it can lead to level performance and ill health.

The amount of stress a person can withstand depends very much on the individual. Some people thrive on stress, and such characters are obviously prime material for managerial responsibilities. Others crumple at the sight of unusual difficulties.

When exposed to stress, in whatever form, we react both chemically and physically. In fact, we invoke the “fight” mechanism, which in more primitive days made the difference between life and death. The crises we meet today are unlikely to be so extreme, but however minimal the stress, it involves the same response. All the energy is diverted to cope with the stress, with the result that other functions, such as digestion, are neglected.

It is when such a reaction is prolonged, through continued exposure to stress, that health becomes endangered. Such serious conditions as high blood pressure, coronary heart disease(冠心病)all have established links with stress. The way stress affects a person also varies with the individual. Stress in some people produces stomach disorders, while others succumb to tension headaches. Since we cannot remove stress from our lives, we need to find ways to cope with it.

1.The reason that many people find it very difficult to relax these days is that ___.
A.they are working harder than they used to be.
B.they are often too busy to find the time.
C.they are suffering from the effects of stress.
D.they are not clear of how to relax by themselves.

2.We learn from the passage that ___.
A.how much stress one can bear depends greatly on whether he knows the art of relaxation.
B.people in primitive days survived from stress because they found certain mechanism to cope with it.
C.if one gets into the habit of relaxing every day he can overcome stress easily.
D.stress can lead to serious health problem if one is exposed to it for too long.

3.The sentence “Stress, in fact, is not the ‘baddy’ it is often reputed to be” suggests that ___.
A.stress used to have a bad reputation of causing ill health.
B.we should not take it for granted that stress is unavoidable.
C.stress is not so terrible as people often believe it to be.
D.people do not think stress is as harmful as it was before.

4.The pronoun “it” at the end of the passage refers back to __.
A.ill health
B.exposure
C.reaction
D.stress.

5.What is writer’s attitude to stress according to the passage?
A.Stress as well as relaxation is essential for a healthy mind and body.
B.Stress produces both positive and negative effects on people.
C.Stress should not be eliminated completely from the life.
D.People usually work better under stress if they are healthy.

第30篇答案:BDCDB

TOP

返回列表